asda released papers l-m

46

Upload: zuhair-waheed

Post on 11-Mar-2015

1.058 views

Category:

Documents


17 download

TRANSCRIPT

Page 1: ASDA Released Papers L-M
Page 2: ASDA Released Papers L-M

22 201. The posterior belly of the digastric is supplied

by which of the following nerves?

A. Vagus B. Facial C. Trigeminal D. Cervical spinal E. Inferior alveolar

202. Which of the following pairs of amino acids is expected to be found on the interior of a globular protein?

A. Lysine and arginine B. Arginine and leucine C. Leucine and valine D. Valine and glutamic acid E. Glutamic acid and lysine

203. Which of the following conditions might be an initial manifestation of early, acute HIV infection?

A. Kaposi's sarcoma B. Wasting syndrome C. Oral hairy leukoplakia D. Mononucleosis-like syndrome E. Pneumocystis carinii pneumonia

204. Which of the following primary teeth has a distinctly prominent facial cervical ridge that makes it different from other teeth?

205.

A. Maxillary canine B. Mandibular canine C. Mandibular first molar D. Maxillary second molar E. Maxillary central incisor

An anemic individual is expected to have which of the following conditions in systemic arterial blood?

Oxygen tension Oxygen content

A. Elevated Normal

B. Elevated Reduced

C. Reduced Normal

D. Normal Reduced E. Reduced Reduced

206. The horizontal plane diagram below shows each of the following components of mandibular movement EXCEPT one. Which one is this EXCEPTION?

207.

A.

B.

c.

D.

E.

Vertical component of the movement frorr retruded contact position (1) to intercuspal position (2) Horizontal component of movement from retruded contact position (1) to intercuspal position (2) Anterior component of movement from intercuspal position (2) to maximum protrusive position (4) Lateral component of movement from retruded contact position (1) to intercuspal position (2) Mandibular movements occurring laterotrusively

In the presence of sucrose, Streptococcus mutans produces

A. B. C. D.

E.

L-forms. very long chains of cocci. regular diploid pairs rather than chains. deposits of a gummy polysaccharide called glucan. a very thick wall of muramic acid and mucopeptide substances.

208. Which of the following properties do mast cells and basophils have in common?

A. Found in blood B. Develop from lymphocytes C. Secrete heparin and histamine D. Decrease during time of infection E. Migrate to line the inside of blood

vessels

209. Protein and RNA synthesis occur in each of the following phases of the cell cycle EXCEPT one. Which one is this EXCEPTION?

A. Go

B. G,

C. S D. G2

E. M

© 2005 American Dental Association, Joint Commission on National Dental Examinations. All rights reserved. Reprinted by permission.

Page 3: ASDA Released Papers L-M

210. Each of the following is attributable to hepatic failure EXCEPT one. Which one is this EXCEPTION?

211.

212.

A. B. C. D. E.

Tremor Gynecomastia Mallory bodies Hypoalbuminemia Spider telangiectasia

As viewed from the lingual, the lingual cusp of a maxillary first premolar is inclined

distally. mesially. d istofacially. mesiolingually.

.... A. B. C. D. E. directly over the center mesiodistally.

Gram-positive cell walls differ from gram­negative cell walls in that gram-positive cell walls uniquely contain

A. teichoic acids. B. lipopolysaccharides. C. acid-fast lipids. D. endotoxins.

213. The main route of calcium excretion from a normal human adult is

A. urine. B. sweat. C. saliva. D. feces. E. tears.

214. The parotid duct crosses the

A. temporalis. B. facial artery. C. masseter muscle. D. zygomaticus major. E. lateral pterygoid.

215. The cusp of Carabelli occurs with sufficient frequency to be considered normal in which of the following molars?

A. Primary maxillary first B. Primary maxillary second C. Mandibular first D. Maxillary second

216. Which of the following temporomandibular joint ligaments restricts the movement of the disc away from the condyle during.function?

A. B. C. D.

Discal Stylomandibular Temporomandibular Sphenomandibular

217. A patient who has anemia, poorly localized abdominal pain, and wrist and foot drop probably is manifesting a toxic state induced by which of the following?

A. Heroin B. Cocaine C. Bismuth D. Mercury E. Lead

218. Which of the following is a ketone body?

219.

220.

A. Glycerol B. Glucagon C. Acetyl CoA 0, Acetoacetate E. Phosphatidylcholine

Inorganic crystals in enamel have their long axes parallel to the rods in which of the following?

A. Centers of the bodies of the rods only B. Periphery of the bodi.es of tbe rops

only C. Bodies of the rods and deviating

increasingly in the tails D. Tails of the rods and deviating

increasingly from the centers of the bodies to the periphery

E. Tails of the rods and deviating increasingly from the periphery of the bodies to the centers

The occlusal view of a normal dental arch segment is shown below. The arrow points to which of the following mandibular teeth?

A. Primary right first molar B. Primary left first molar C. Permanent right first premolar D. Permanent left second premolar

© 2005 American Dental Association, Joint Commission on National Dental Examinations. All rights reserved. Reprinted by permission.

23

Page 4: ASDA Released Papers L-M

22

201. The posterior belly of the digastric is supplied by which of the following nerves?

A, Vagus g, Facial C, Trigeminal D, Cervical spinal E, Inferior alveolar

202. Which of the following pairs of amino acids is expected to be found on the interior of a globular protein?

A, Lysine and arginine B. Arginine and leucine C, Leucine and valine D, Valine and glutamic acid E. Glutamic acid and lysine

203. Which of the following conditions might be an initial manifestation of early, acute HIV infection?

A, Kaposi's sarcoma B. Wasting syndrome C, Oral hairy leukoplakia D, Mononucleosis-like syndrome E, Pneumocystis carinii pneumonia

204. Which of the following primary teeth has a distinctly prominent facial cervical ridge that makes it different from other teeth?

205.

A. Maxillary canine g, Mandibular canine C, Mandibular first molar D, Maxillary second molar E. Maxillary central incisor

An anemic individual is expected to have which of the following conditions in systemic arterial blood?

Oxygen tension Oxygen content

A. Elevated Normal g, Elevated Reduced C, Reduced Normal D, Normal Reduced E. Reduced Reduced

206. The horizontal plane diagram below shows each of the following components of mandibular movement EXCEPT one, Which one is this EXCEPTION?

207.

A.

B,

c,

D,

E.

Vertical component of the movement fror retruded contact position (1) to intercuspal position (2) Horizontal component of movement from retruded contact position (1) to intercuspal position (2) Anterior component of movement from intercuspal position (2) to maximum protrusive position (4) Lateral component of movement from retruded contact position (1) to intercuspal position (2) Mandibular movements occurring laterotrusively

In the presence of sucrose, Streptococcus mutans produces

A, B. C, D.

E.

L-forms. very long chains of cocci. regular diploid pairs rather than chains. deposits of a gummy polysaccharide called glucan. a very thick wall of muramic acid and mucopeptide substances.

208. Which of the following properties do mast cells and basophils have in common?

A. Found in blood B. Develop from lymphocytes C. Secrete heparin and histamine D, Decrease during time of infection E. Migrate to line the inside of blood

vessels

209. Protein and RNA synthesis occur in each of the following phases of the cell cycle EXCEPT one, Which one is this EXCEPTION?

A, Go

B, G,

C, S

D. G2

E. M

© 2005 American Dental Association, Joint Commission on National Dental Examinations. All rights reserved. Reprinted by permission,

Page 5: ASDA Released Papers L-M

24 221. The presence of mamelons in a 10-year-old

patient suggests which of the following conditions?

A. B. C. D. E.

Fluorosis Malnutrition Malformation Anterior open bite Retained primary teeth

222. Which of the following pathogens is MOST often associated with infective endocarditis?

A. Pneumococci B. Group A streptococci C. Group B streptococci D. a-hemolytic streptococci E. ~-hemolytic streptococci

223. Which of the following coenzymes is required for the synthesis of pyrimidines, such as thymine?

A. B. C. D. E.

Niacin Thiamine Folic acid Riboflavin Pyridoxine

224. Each of the following venous channels has direct connections with the pterygoid venous plexus EXCEPT one. Which one is this EXCEPTION?

A. B. C. D. E.

Maxillary vein Vertebral vein Deep facial vein Infraorbital vein Posterior superior alveolar vein

225. Vitamin C deficiency primarily affects which of the following tissues?

A. B. C. D. E.

Hematopoietic Connective Epithelial Muscular Nervous

226. In the upper limb, which of the following represents a hallmark of lymphatic vessels?

A. B. C. D.

E.

Contain valves Follow the veins Always travel in pairs Only found on the anterior surface of the limb Contain fenestrations to allow passage of fluids into the interstitium

227. When viewed from the incisal aspect, each of the following features of an incisor is visible EXCEPT one. Which one is this EXCEPT/ON?

228.

A. Cingulum B. Lingual fossa C. Cervical line D. Marginal ridges E. Mesiofacial developmental depression

Each of the following is expected to be active during fatty acid biosynthesis EXCEPT one. Which one is this EXCEPT/ON?

A. Tricarboxylic acid cycle B. Pyruvate dehydrogenase C. Amino acid catabolism D. Beta oxidation E. Glycolysis

229. Pain in inflamed tissues is associated with the release of which of the following mediators?

A. B. C. D. E.

Histamine Serotonin Bradykinin Leukotrienes Complement components

230. Which of the following cranial nerves contains parasympathetic components?

A. B. C. D. E.

Facial Abducens Trigeminal Hypoglossal Spinal accessory

© 2005 American Dental Association J . t c .. N' I D '" . . . ,Oln ommlsslon on atlona ental Examinations. All nghts reserved. Repnnted by permission.

Page 6: ASDA Released Papers L-M

231. In an ideal intercuspal position, the mesiofacial groove of a mandibular first molar opposes which part of the maxillary first molar?

232.

233.

234.

A. B. C. D.

Tip of the distofacial cusp Tip of the mesiofacial cusp Triangular ridge of the distofacial cusp Triangular ridge of the mesiofacial cusp

Which of the following represents genes that often encode proteins necessary for antibiotic resistance, and that can change positions on a chromosome or "jump" from a plasmid to a chromosome?

A. Tryptophan operon B. Lysogens C. Prophage D. Transposons

Which of the following BEST describes the major function of histones?

A. Unwind DNA. B. Activate genes. C. Stabilize DNA in a compact form. D. Remove exons during RNA splicing. E. Stabilize RNA during transcription.

The thyrocervical trunk is comprised of the inferior thyroid, the transverse cervical artery, and the

A. B. C. D. E.

vertebral. suprascapular. dorsal scapular. superior thyroid. ascending pharyngeal.

235. Under physiologic conditions, the LOWEST partial pressure of oxygen is found in

236.

A. expired air. B. alveolar air. C. venous blood. D. arterial blood. E. normal atmosphere air.

The condyle on the laterotrusive side generally rotates about a

A. sagittal axis only. B. horizontal axis only. C. horizontal axis and translates laterally. D. vertical axis and translates later01ly.

237. Each of the following diseases is transmitted from person to person by droplets or droplet spray EXCEPT one. Which one is this EXCEPT/ON?

238.

239.

A. B. C. D. E.

Condyloma acuminatum Whooping cough Meningitis Diphtheria Pneumonia

Which of the following bones forms the roof of the orbit?

A. Zygomatic B. Maxilla C. Palatine D. Sphenoid E. Frontal

Which of the following describes sebaceous glands of the skin?

A. They are associated with hair follicles and are derived from ectoderm.

B. They serve a secretory function in conjunction with Pacinian corpuscles.

C. They secrete a serous material by the merocrine method.

D. They yield a salty secretion and are derived from mesoderm.

240. The first step in the pathway for fatty acid synthesis is catalyzed by an allosteric enzyme which is the principal regulator of the pathway. This enzyme is

A. thiolase. B. pyruvate carboxylase. C. citrate synthetase. D. acetyl CoA carboxylase. E. pyruvate dehydrogenase.

241. Which of the following is a major disadvantage of ethylene oxide sterilization?

A. B. C. D. E.

Difficult to obtain Very toxic to humans Requires high temperatures (160°) Causes corrosion of metal instruments Causes discoloration of clear plastic objects

© 2005 American Dental Association, Joint Commission on National Dental Examinations. All rights reserved. Reprinted by permission.

25

Page 7: ASDA Released Papers L-M

26 242. What teeth occlude with only one opposing

tooth?

A. Maxillary canines B. Mandibular lateral incisors C. Mandibular central incisors D. Maxillary first premolars E. Mandibular third molars

243. Endochondral ossification occurs in the formation of

A. B. C. D. E.

all bones. flat bones. long bones. elastic cartilage. mesenchymal sheets.

~

244. Which of the following represents the MOST common cause of bronchiolitis and pneumonia in infants?

A. Measles virus B. Cytomegalovirus C. Influenza virus D. Herpes simplex virus E. Respiratory syncytial virus

245. The developmental groove between the distofacial cusp and the distal cusp of a mandibular first molar is the

A. B. C. D. E.

facial. distal. distofacial. mesiofacial. distal oblique.

246. Which of the following represents the anterior boundary of the mandibular fossa of the temporal bone?

A. Styloid process B. Articular eminence C. Petrotympanic fissure D. External auditory meatus E. Mastoid process of temporal bone

247. The development of which of the following types of hepatitis depends on the host having been previously infected with hepatitis B virus?

A. Hepatitis A B. Hepatitis C C. Hepatitis D D. Hepatitis E E. Hepatitis G

248. Glucose-6-phosphatase is found associated wifh which of the following?

A. B. C. D.

Kidneys and liver Liver and muscles Kidneys and brain Kidneys and muscles

249. The osteoclast originates from which of the following?

A. Monocyte B. Osteocyte C. Osteoblast D. Neutrophil E. Bone-lining cell

250. Which of the following permanent roots is thin mesiodistally, wide faciolingually, and concave on both mesial and distal surfaces?

A. B. C. D. E.

Maxillary central incisor Mandibular central incisor Mandibular second premolar Facial root of a maxillary first premolar Lingual root of a maxillary first molar

251. The catabolism of which of the following results in no energy production in the form of ATP?

A. Lipid B. Protein C. Nucleotide D. Carbohydrate

252. Recovery from an infection with mumps virus confers lifelong immunity. This describes which of the following types of immunity?

A. B. C. D. E.

Innate Natural active Natural passive Arlificial active Arlificial passive

© 2005 American Dental Association, Joint Commission on National Dental Examinations. All rights reserved. Reprinted by permission.

Page 8: ASDA Released Papers L-M

253. Which of the following represents the structure in the maxillary alveolar bone that maxillary premolar roots occasionally penetrate?

254.

A. B. C. D. E,

Antrum Nasal septum Frontal sinus Zygomatic arch Mandibular fossa

The mylohyoid ridge (line) is found on which of the following portions of the mandible?

A. Coronoid B. Neck C. Ramus D. Condyle E. Body

255. Which of the following represents the portion of the bacterial endotoxin molecule that is MOST responsible for its toxic activity?

A. B. C. D.

Lipid A Protein A o antigen Core polysaccharide

256. Which of the following BEST describes restriction enzymes?

A. B. C. D. E.

Exonucleases Topoisomerases Enzymes that degrade RNA Non-specific endonucleases Site-specific endonucleases

257. The contact areas of anterior teeth are incisal to the middle thirds in each of the following EXCEPT one. Which one is this EXCEPTION?

A. B. C.

D.

E.

Distal contact of the maxillary canine Mesial contact of the maxillary canine Mesial contact of the maxillary lateral incisor Mesial contact of the mandibular lateral incisor Distal contact of the mandibular-lateral incisor

258. Proteins for extracellular use can be synthesized in which of the following structu res?

A. B. C. D. E.

Nucleolemma Mitochondria Heterochromatin Rough endoplasmic reticulum Smooth endoplasmic reticulum

259. Which of the following types of blotting can be used to identify DNA restriction fragments?

260.

A. Eastern B. Southern C. Northern D. Western

The unique amino acid composition of collagen is reflected in the high content of

A. B. C. D. E.

valine, threonine, and lysine. desmosine, glycine, and proline. cysteine, tyrosine, and phenylalanine. glycine, proline, and hydroxyproline. cysteine, hydroxylysine, and proline.

261. Which of the following represents the type of maxillofacial bone graft having the BEST chance of success?

A. Isograft B. Homograft r- Allograft v.

D. Autograft E. Xenograft

262. Which of the following represents a risk factor for the development of endometrial carcinoma?

A. B. C. D. E.

Herpes simplex virus infection Endometriosis Multiple leiomyomas Multiple pregnancies Hyperestrogenism

263. Which of the following vitamins is essential for the normal elaboration and maintenance of bone matrix, cartilage, and dentin?

A. B. C. D.

Niacin Vitamin E Ascorbic acid Pantothenic acid

© 2005 American Dental Association, Joint Commission on National Dental Examinations. All rights reserved. Reprinted by permission.

27

Page 9: ASDA Released Papers L-M

28 264. Calcium stimulates muscle contraction. Its

effect is mediated through binding to which of the following?

A. B. C. D. E.

Actin Tropomyosin Troponin I Troponin C Sarcoplasmic reticulum

265. Which of the following features helps to distinguish a maxillary lateral incisor from a mandibular lateral incisor? The maxillary lateral has

266.

267.

A. a thinner incisal ridge. B. a crown more twisted on the root. C. a more pronounced lingual fossa. D. a root that is flattened mesiodistally. E. a mesioincisal angle that is sharper.

The arrow in the illustration below represents the path taken by an opposing cusp. Which of the following is the cusp and mandibular movement indicated by the arrow?

A.

B.

c.

D.

Facial cusp of a second premolar, retrusive Mesiofacial cusp of a first molar, protrusive Mesiofacial cusp of a second molar, laterotrusive Distofacial cusp of a first molar, protrusive

Which of the following produce surfactant?

A. Monocytes B. Clara cells C. Goblet cells D. Type I pneumocytes E. Type" pneumocytes

268. Each of the following fluids is considered one that can transmit HIV EXCEPT one. Which one is this EXCEPTION?

A. B. C. D. E.

Semen Serum Saliva Amniotic fluid Breast milk

259. When compared to a maxillary first molar, the roots of a maxillary second molar

A. B. C. D. E.

are longer. are more divergent. are fewer in number. have less potential for fusion. are greater in distal inclination.

270. In right laterotrusive movement, the lingual cusp of a maxillary right second premolar passes through which of the following mandibular structures?

A. B. C.

D.

Facial groove of the right first molar Lingual groove of the right first molar Embrasure between the right first premolar and the right second premolar Embrasure between the right second premolar and the right first molar

271. Which of the following is the MOST appropriate sterilization method for a heatlabile item to be used in a surgical procedure?

272.

A. Dry heat B. Autoclaving C. Ethylene oxide D. Saturated chemical vapor E. Immersion in chemical sterilant

The primary function of the anterior portion of the temporalis muscle is to

A. B. C. D. E.

elevate the mandible only. protrude the mandible only. elevate and retrude the mandible. help stabilize the articular disc. move the mandible to the contralateral side.

273. Bile salts are sodium salts of bile acid that are conjugated with

A. B. C. D.

cysteine and serine. taurine and glycine. methionine and lysine. homocysteine and ornithine.

© 2005 American Dental Association, Joint Commission on National Dental Examinations. All rights reserved. Fleprinted by permission.

Page 10: ASDA Released Papers L-M

274.

275.

276.

277.

278.

In order for RNA oncogenic viruses to be integrated into the host genome, they must possess

A. translation enzymes. B. restriction endonuclease. C. DNA-dependent RNA polymerase. D. RNA-dependent DNA polymerase.

On the lingual crown surface of maxillary canines, which of the following is the structure located immediately mesial to the distolingual fossa?

A. Cingulum B. Lingual ridge C. Mesiolingual fossa D. Mesial marginal ridge E. Distal marginal ridge

In which of the following structures are glycoproteins assembled for extracellular use?

A. Nucleolemma B. Polyribosomes C. Golgi apparatus D. Rough endoplasmic reticulum E. Smooth endoplasmic reticulum

Which of the following statements BEST describes the strength of cardiac muscle contraction? It is

A. decreased when extracellular Ca++ is increased.

B. increased when extracellular Ca++ is increased.

C. mainly regulated by vagal impulses. D. unaffected by extracellular Ca++' E. mainly regulated by heart rate.

Oxygen-dependent killing of bacteria by neutrophils involves each of the following EXCEPT one. Which one is this EXCEPTION? A. Superoxide B. Collagenase C. Myeloperoxidase D. Hydrogen peroxide E. NADP dehydrogenase

279.

280.

281.

282.

283.

29

Which of the following are the teeth whose masticatory function is primarily biting?

A. Incisors and canines B. Incisors and premolars C. Canines and premolars D. Premolars and molars

Which of the following represents the predominant serum immunoglobulin found during a memory response?

A. IgA B. IgD C. IgE D. IgG E. s-lgA

Which of the following mechanisms operates in liver cells to regulate breakdown of glycogen?

A. Zymogen activation B. Phosphorylation of phosphorylase b to

phosphorylase a C. Induction of phosphorylase b by an

inducer . D. Inhibition of adenylate cyclase

Which of the following structures can be palpated by way of the external auditory (acoustic) meatus?

A. Zygomatic arch B. Mandibular notch C. Lateral pterygoid muscle D. Coronoid process of the mandible E. Posterior aspect of the mandibular

condyle

Which main product of protein nitrogen metabolism is found in human urine?

A. Urea B. Ammonia C. Creatine D. Uric acid E. Creatinine

© 2005 American Dental Association, Joint Commission on National Dental Examinations. All rights reserved. Reprinted by permission.

Page 11: ASDA Released Papers L-M

30 284. Which of the follm·ving nerves travels from the

carotid triangle into the submandibular triangle of the neck?

A. Lingual B. Hypoglossal C. Great auricular D. Ansa cervicalis E. Recurrent laryngeal

285. Which of the following best represents the structure on the crown of the maxillary canine which normally lies directly distal to the mesiolingual fossa?

A. B. C. D. E.

Lingual ridge Distal fossa Distolingual fossa Distal marginal ridge Mesial marginal ridge

286. Which of the following organisms causes a granulomatous, tuberculosis-like infection?

A. B. C. D. E.

Aspergillus fumigatus Microsporum audouini Histoplasma capsula tum Epidermophyton floccosum Trichophyton schoenleinii

287. Stimulation of the posterior hypothalamus by a reduction in core temperature will produce which of the following?

288.

A. Panting B. Sweating C. Vomiting D. Shivering E. Vasodilation

Each of the following structures is found in the posterior mediastinum EXCEPT one. Which one is this EXCEPTION?

A. B. C. D. E.

Vagus nerve Phrenic nerve Thoracic duct Greater splanchnic nerve Sympathetic chain ganglia

289. When an adult with normal occlusion moves the mandible from maximum intercuspation to right lateral relation, which of the following cusps of maxillary left teeth moves through the facial embrasure between the mandibular left premolars?

Cusp Tooth

A. Lingual First premolar B. Facial Second premolar C. Lingual Second premolar D. Mesiofacial First molar E. Distofacial First molar

290. A patient has a periapical abscess and a history of hypersensitivity to penicillin. Which of the following antimicrobial agents might be the preferred alternative for this patient?

A. B. C. D. c L.

Amoxicillin Azith romycin Tetracycline Cephalosporin Nystatin

291. Which of the following compounds is a precursor of the pigments of the skin, hair, and eyes?

A. B. C. D. E.

Tyrosine Cysteine Lysine Creatine Galactosamine

292. Intelligence and sensory-motor functions of a patient appear to be intact. However, the patient lacks self-discipline and has exhibited anti-social behavior. He is unable to plan for the future or to organize behaviors into logical sequences. A lesion in which of the following areas is likely?

A. B. C. D. E.

Frontal lobe Occipital lobe Internal capsule Temporal lobe Parietal lobe

© 2005 American Dental Association, Joint Commission on National Dental Examinations. All rights reserved. Reprinted by permission.

Page 12: ASDA Released Papers L-M

293. Which of the following organs secretes insulin­like growth factor 1 (IGF-1), and what stimulates this secretion?

294.

295.

296.

A.

B.

Anterior pituitary, increasing IGF-RH in the blood Anterior pituitary, increasing GRH in the blood

C. Kidney, increasing growth hormone in the blood

D. Liver, increasing growth hormone in the blood

E. Liver, increasing TSH in the blood

Edema as a result of a hemodynamic disorder commonly occurs in each of the following locations EXCEPT one. Which one is this EXCEPTION?

A. Lungs B. Brain C. Subcutaneous tissues D. Peritoneal cavity E. Pancreas

The tissue present between the articulating bones of the temporomandibular joint is called the

A. B. C. D. E.

articular disc. fibrous capsule. synovial membrane. joint capsule. bilaminar zone.

The progression from a healthy gingival sulcus to gingivitis is associated with a shift towards

A. gram-positive cocci. B. gram-negative anaerobic rods. C. gram-positive filamentous rods. D. gram-negative anaerobic cocci. E. gram-positive facultative cocci.

297. Which of the following enzymes synthesizes polynucleotide chains from nucleotides and does NOT require a primer chain?

298.

299.

A. B. C. D. E.

Ligase Exonuclease Endonuclease DNA polymerase RNA polymerase

Which of the following structures travels just posterior to the arch of the azygos vein?

A. Hemiazygos vein B. Right vagus nerve C. Left vagus nerve D. Right lymphatic duct E. Greater splanchnic nerve

An infection spreading by way of the lymphatic system from the vermiform appendix first enters the blood stream at the

A. B. C. D. E.

renal vein. appendicular vein. inferior vena cava. brachiocephalic vein. internal thoracic vein.

300. Which of the following describes a major effect of sickle cell anemia?

A.

B. C.

D.

E.

Absence of biphosphoglycerate binding of hemoglobin Substitution of 2 proximal histidines Decreased solubility of the deoxy form of hemoglobin A P50 value for hemoglobin similar to that

of myoglobin Decreased number of subunits in hemoglobin

© 2005 American Dental Association, Joint Commission on National Dental Examinations. All rights reserved. Reprinted by permission.

31

Page 13: ASDA Released Papers L-M

301. Human infection with Histoplasma capsula tum is usually a result of

302.

303.

304.

A. B. C. D. E.

inhaling contaminated air. drinking contaminated water. ingesting contaminated food. direct human-to-human contact. direct animal-to-human contact.

From a frontal view, which of the following describes the plane of occlusion of the mandibular arch in a normal dentition?

A. It is flat. B. It is a convex curve. C. It is a concave curve. D. It is the result of uniform perpendicular

alignment of the teeth.

An abnormal increase in blood pressure in a healthy person will RESUL Tin an increased number of impulses traveling to the heart ove!" which of the following nerves?

A. Vagus nerves B. Sympathetic nerves C. Branches of thoracic nerves D. Branches of the trigeminal nerve E. Branches of the glossopharyngeal

nerve

Which of the following hormones exerts the LEAST effect on calcium metabolism of bone tissue?

A. B. C. D. E.

Androgen Estrogen Norepinephrine Thyroid hormone Parathyroid hormone

305. Which of the following types of bronchogenic carcinoma is MOST commonly associated with paraneoplastic syndrome?

A. B. C. D.

Small cell carcinoma Large cell carcinoma Mesothelioma Adenocarcinoma

306. Which of the following teeth normally has the longest root?

A. B. C. D. E.

Maxillary canine Mandibular canine Maxillary central incisor Mandibular first premolar Maxillary second premolar

307. Tooth erosion in bulimic patients is due to

308.

309.

A. hyposalivation. B. hypersalivation. C. action of pepsin. D. excessive fluoride intake. E. solubility of hydroxyapatite in acid.

Which of the following represents the basis for the topical application of fluorides in dental caries prevention?

A.

B.

C.

D.

E.

Fluoride penetrates the enamel through the lamellae. Keratin content of the enamel is made more insoluble. Fluoride coats the tooth forming a uniform protective barrier. The primary cuticle. being less calcified, absorbs the fluoride. Acid solubility of the surface enamel is reduced by the fluoride.

Which of the following statements describes an exampl0 of innate immunity?

A. B. C.

D.

E.

An allergic reaction to insect venom The classical pathway of complement The destruction of virus-infected cells by T-killer cells The production of IgG in response to insect venom Phagocytosis of microbes by neutrophils and macrophages

310. Which of the following mandibular molar proximal crown surfaces normally exhibits the LARGEST size?

A. B. C. D. E.

Mesial of the first Distal of the first Mesial of the second Distal of the second Mesial of the third

© 2005 American Dental Association, Joint Commission on National Dental Examinations. All rights reserved. Reprinted by permission.

Page 14: ASDA Released Papers L-M

311.

312.

In a CUSP TO FOSSA contacting relationship in intercuspal position, the maxillary first premolar is MOST likely to articulate with which of the following mandibular teeth?

A. Lateral incisor B. Lateral incisor and canine C. First premolar D. Canine and first premolar E. First premolar and second premolar

Which of the following has the LEAST ability to regenerate?

A. B. C. D. E.

Bone Liver Striated muscle Collagen Smooth muscle

313. Which of the following represents normal glucose clearance?

314.

315.

316.

A. 0 mg/min B. 44 mg/min C. 125 mg/min D. 600 mg/min

There is a distinct change in the type of surface epithelium at the junction of the

A. ileum and colon. B. fundus and pylorus. C. duodenum and jejunum. D. stomach and esophagus. E. oropharynx and esophagus.

A patient bites down rapidly on an unexpected hard surface while chewing. Cessation of motor unit recruitment in jaw closing muscles is caused by stimulation of

A. muscle spindles. B. mucosal mechanoreceptors. C. periodontal mechanoreceptors. D. nociceptors in the dental pulp.

Pulmonary edema and filling of the pleural cavity by a transudate is MOST suggestive of which of the following underlying diseases?

A. Cor pulmonale B. Pulmonary embolism C. Pulmonic valve stenosis D. Tricuspid valve prolapse E. Left-sided heart failure

317.

318.

The major grooves of the mandibular second molar form which of the following patterns?

A. H B. Y C. X D. + E.

If the maxillary and mandibular incisors are in crossbite, then the contacting surfaces, during maximum intercuspation, would be

A. B. C.

D.

mandibular lingual and maxillary facial. mandibular facial and maxillary facial. mandibular lingual and maxillary lingual. mandibular facial and maxillary lingual.

319. Which of the following disorders is X-linked recessive?

320.

321.

A. B. C. D. E.

Neurofibromatosis Turner syndrome Cystic fibrosis Marfan syndrome Hemophilia A

Active transport systems generally involve specific binding molecules that are classified as a

A. lipid. B. protein. C. lipoprotein. D. carbohydrate. E. phospholipid.

Which of the following structures directly develops from the cells remaining in the remnants of the preovulatory follicle after ovulation?

A. Theca interna B. Corpus luteum C. Corpus albicans D. Atretic follicle E. Interstitial gland

© 2005 American Dental Association, Joint Commission on National Dental Examinations. All rights reserved. Reprinted by permission.

33

Page 15: ASDA Released Papers L-M

34 322. Which of the following provides the immediate

energy source for the transport of glucose into intestinal epithelial cells?

A. B. C.

D.

NADH NADPH Na+ gradient across the luminal membrane H+ gradient across the luminal membrane

323. The biologic effect of exotoxin A produced by Pseudomollus aeruginosa is to

324.

325.

326.

A. increase membrane permeability, B. inactivate IgG. C. inhibit protein synthesis. D. increase levels of cAMP. E. inhibit DNA synthesis.

As distinguished from a maxillary canine, a mandibular canine typically has

A. a sharper facial cusp tip. B. contact areas located more incisally. C. a more pronounced cingulum. D. a more convex mesial border, when

viewed facially.

Urolithiasis is associated with each of the following conditions EXCEPT one. Which one is this EXCEPTION?

A. Gout B. Hypertension C. Hypercalcemia D. Renal infection E. Hyperparathyroidism

An action potential initiated at the midpoint along the length of an axon will

A. not propagate. B. propagate towards the soma. C. propagate towards the nerve ending. D. propagate towards both the soma and

the nerve ending.

327. In which of the following are tonofibrils and desmosomes especially well-developed?

A. Neurons

B. Fibroblasts

C. Ameloblasts

D. Odontoblasts

E. Keratinocytes

328. Which of the following cells are responsible for delayed-type hypersensitivity (Type IV hypersensitivity) ?

329.

330.

331.

332.

A. B. C. D. E.

B cells, Th2 cells B cells, neutrophils Th1 cells, macrophages Mast cells, eosinophils IgE, mast cells, histamine

Which of the following teeth has its mesial marginal ridge located more cervically than its distal marginal ridge?

A. Mandibular first premolar B. Maxillary first premolar C. Maxillary first molar D. Primary mandibular first molar E. Mandibular second molar

Which of the following represents the ridge on the occlusal surface of the maxillary first molar that normally forms the distal boundary of the central fossa?

A. Oblique B. Distal marginal C. Mesial marginal D. Mesial cusp ridges of the MF cusp E. Distal cusp ridges of the OF cusp

Which of the following characterizes a benign tumor arising from skeletal muscle?

A. Leiomyoma B. Papilloma C. Rhabdomyoma D. Leiomyosarcoma E. Rhabdomyosarcoma

The dentin of the crown of a tooth and that of the root differ in which of the following ways?

A. B. C.

D.

E.

Root dentin contains lacunae. Root dentin is more highly calcified. Root dentin contains more tubules per given area. Granular layer of Tomes is found primarily in the crown. Primary curvatures of the tubules decrease in root dentin.

© 2005 American Dental Association, Joint Commission on National Dental Examinations. All rights reserved. Reprinted by permission.

Page 16: ASDA Released Papers L-M

333. Which of the following explains why an increased filling of the ventricle during diastole causes a more forceful heart beat? The increased ventricular volume

A.

B. C. D.

diminishes the refractory period of the ventricle. increases end-diastolic fiber length. facilitates conduction in the heart. produces a less than optimal load.

334. Following prophylactic administration of amoxicillin, a patient becomes hypotensive and itchy and is having difficulty breathing. Which of the following is MOST probably occurring?

335.

336.

A. CD4 lymphocytes are secreting Iymphokines resulting in edema.

B. Amoxicillin reacts with IgE and activates cytotoxic T cells to release Iymphokines.

C. IgG and complement are inducing the chemotaxis of neutrophils out of vessels.

D. IgE bound to antigen results in histamine release from mast cells,

A transverse ridge results from the union of

A. Mesial and distal marginal ridges B. Facial and lingual cusp ridges C. Facial and lingual triangular ridges D. Marginal and triangular E. Cusp ridge and triangular ridge

Which of the following is MOST likely to precede an impending atherosclerotic cerebral infarction?

A. Angina pectoris B. Pulmonary embolus C. Myocardial infarction D. Ruptured berry aneurysm E. Transient ischemic attacks

337. The biologically active conformation of trimeric G-proteins requires

A. the alpha-subunit to bind GDP. B. the alpha-subunit to bind GTP. C. the hydrolysis of BetaGamma-subunits. D. the alpha-subunit to phosphorylate

downstream targets.

338. During the life span of a multi rooted tooth, dentin continues to form MOST rapidly at which of the following locations?

A. B. C. D.

E.

Within the root canals At the dentinoenamel junction Floor and roof of the pulp chamber Mesial and distal walls of the pulp chamber Facial and lingual walls of the pulp chamber

339. The glomerular lesions in immune complex disease result from deposition of which of the following?

A. IgE B. IgG C. TCR D. Histamine E. Interleukin

340. Which of the following is normally exhibited by maxillary second premolars?

A. Two roots B. A mesial concavity of the crown C. A facial crown contour that is greatly

inclined to the lingual D. A lingual height of contour in the

cervical third of the crown E. A crown that is wider faciolingually

than mesiodistally

341. The articular disc of the temporomandibular joint consists of

A. 8. C.

D. E.

hyaiine cartilage. loose fibrous connective tissue. an outer synovial layer and an inner fibrous layer. dense fibrous connective tissue. an outer layer of mesothelium and an inner core of hyaline cartilage.

342. Which of the following diseases is caused by an agent that produces neither exotoxins nor endotoxins?

A. Cholera B. Tetanus C. Brucellosis D. Gas gangrene E. Tuberculosis

© 2005 American Dental Association, Joint Commission on National Dental Examinations. Ali rights reserved. Reprinted by permission.

35

Page 17: ASDA Released Papers L-M

36 343. Which of the following teeth might possess

three cusps?

A.

B.

C.

D.

E.

Maxillary second premolar and maxillary first molar Maxillary second premolar and mandibular first premolar Maxillary first molar and mandibular second molar Maxillary second molar and mandibular first premolar Maxillary second molar and mandibular second premolar

:~44. Arneloblasts will form enamel if which of the following cells are present?

A. 8. C. D. E.

Stellate reticulum Stratum intermedium Langerhans and Merkel Outer enamel epithelium Reduced enamel epithelium

345. A mandibular first molar usually has how many roots and canals?

Roots Canals

A. Two Two B. Two Three C. Three Two D. Three Three

346. In osteomalacia, bones are weak because of the failure of

A. B. C. D. E.

remodeling of bone. bone matrix formation. osteoblast proliferation. bone matrix calcification. interstitial growth of cartilage.

347. The kidney dialysis machine is an example of which of the following mechanical systems?

A. Sorting B. Cotransport C. Countercurrent D. Active transport

348. The OLDEST enamel in a fully erupted first molar is located at the

A. B. C. D.

E.

cervix of the tooth. oral surface of a cusp. mesial and distal surfaces of the tooth. dentinoenamel junction underlying a cusp. dentinoenamel junction underlying a fissure.

349. Which of the following roots MOST likely has two canals?

A. B. C.

D.

E.

Facial root of the maxillary first premolar Palatal root of the maxillary first molar Distofacial root of the maxillary first molar Mesiofacial root of the maxillary first molar Distal root of the mandibular first molar

350. Which of the following represents the term "characteristic of a single disease"?

351.

A. Pathogenic B. Pathologic C. . Idiopathic D. Iatrogenic E. Pathognomonic

Each of the following statements correctly describes similar characteristics of the chemical senses of taste and smell EXCEPT one. Which one is this EXCEPTION?

A. Receptor cells are replaced regularly. B. There are primary classes of taste and

of odor. C.

D.

E.

Receptors are located on cilia or microvilli at the apical ends of cells. Receptors initiate action potentials directly to respective cranial nerve sensory fibers. Molecules must be dissolved in saliva or mucus to interact with receptor membrane proteins.

© 2005 American Dental Association, Joint Commission on National Dental Examinations. All rights reserved. Reprinted by permission.

Page 18: ASDA Released Papers L-M

38

Test item #'5 362-368 refer to the following testlet.

A 42-year-old female with a history of hyperparathyroidism presents with a complaint of bilateral pain in the temporomandibular joint on closing. She reports that her ears are occasionally "clogged", and she sometimes experiences a "ringing" sensation. The dentist determines that these symptoms are related to her TMJ condition.

362. Given her reported hyperparathyroidism, this patient is likely to show each of the following signs or symptoms EXCEPT one. Which one is this EXCEPTION?

A. Urolithiasis B. Elevated serum calcium C. Tetanic muscular convulsions D. Central giant-cell bone lesions E. Loss of lamina dura surrounding

multiple teeth

363. Which of the following will most likely be INCREASED in this patient?

A. Intestinal calcium absorption B. Intestinal organic phosphate absorption C. Renal calcitriol production D. Renal calcium excretion E. Renal organic phosphate excretion

364. This patient will MOST likely be predisposed to which of the following?

A. B. C. D.

Muscle spasm Reduced serum Ca++ levels Increased osteoblastic activity Increased likelihood of bone fracture

365. Examination reveals diminished parotid salivation due to middle ear involvement. Which of the following neNes is MOST likely involved?

A. Facial B. Lesser petrosal C. Greater petrosal D. Auriculotemporal E. Chorda tympanic

366. In this patient, which of the following region of the articular surface of the TMJ is unlikel to be present?

367.

368.

A. Calcified cartilage B. Subarticular bone C. Proliferative zone D. Fibrous arti~ular layer E. Fibrocartilaginous zone

If the patient's sensation of "clogged ears" is due to pressure on the auditory tubes, then which of the following nerves is providing this sensory innervation?

A. Glossopharyngeal B. Lesser occipital C. Vestibulocochlear D. Zygomatic temporal E. Posterior deep temporal

This patient's feeling that her ears are "clogged" might be due to restricted function of which of the following muscles?

A. Digastric B. Omohyoid C. Temporalis D. Buccinator Tensor E. veli palatini

Test item #'s 369-377 refer to the following testlet

A lO-year-old female with a history of acute lymphoblastic leukemia is undergoing orthodontic treatment that is complicated by poor oral hygiene. Oral examination reveals generalized ceNical plaque and enlargement of the gingiva. There is generalized bleeding on probing.

369. Each of the following statements correctly describes a clinical feature of this patient's leukemia EXCEPT one. Which one is this EXCEPTION?

A.

B.

C.

D.

Multiple, acutely tender lymph nodes due to central node ischemic necrosis. CNS infiltration related to headache, vomiting, and palsies. Bone pain secondary to leukemia infiltration of the marrow and periosteum. Symptoms related to bone marrow suppression, including anemia and thrombocytopenia.

© 2005 American Dental Association, Joint Commission on National Dental Examinations. All rights reserved. Reprinted by permission.

Page 19: ASDA Released Papers L-M

40

Test item #'s 378-389 refer to the following testlet.

An 80-year-old female with a history of cardiovascular disease presents with a complaint of pain under her complete dentures when biting. She has worn them for 15 years, but seldom removes or cleans them. Removal of the dentures reveals diffuse erythema of underlying mucosal tissue. Bone resorption is noted from detectable reduction in height of the alveolar ridges due to the ill-fitting denture.

378. Each of the following statements correctly describes myocardial infarction (MI) EXCEPT one. Which one is this EXCEPT/ON?

379.

380.

A.

B.

C.

D.

E.

Most acute Mis are caused by coronary artery th!"Ombosis. Acute MI is the most common cause of death in industrialized nations. Pain f!"Om MI can usually be relieved by vasodilators such as nitroglycerin. Thrombolytic agents such as streptokinase often limit the size of infarction. Myocardial necrosis usually begins 20-30 minutes after coronary artery occlusion.

When fully reclined in the dental chair, the patient reports difficulty in breathing. This is MOST likely the direct result of which of the following?

A. Hypoxia B. Angina pectoris C. Bronchoconstriction D. Carbon dioxide retention E. Congestive heart failure

The atherosclerotic condition of this patient's arteries likely involves hypertrophy of which of the following?

A. B. C. D.

Tunica media Tunica intima Tunica adventitia Internal elastic membrane

381. Each of ihe following statements describes a characteristic feature of ischemic heart disease EXCEPT one. Which one is this EXCEPT/ON?

A

B.

C.

D.

Usually results from complete occlusion of one or more coronary arteries. Represents an imbalance between myocardial oxygen demand and available blood supply. Has a peak incidence in men over 60 years of age, and women over 70 years of age. Contributing factors include chronic hypertension and high levels of low­density lipoproteins.

382. Which of the following represents the MOST likely pathologic change in this patient's heart?

383.

384.

385.

A. Vegetations of the aortic valve B. Severe mitral valve thickening C. Hypert!"Ophy of the left ventricle D. f\lecrosis in the right ventricle E. Scarring in the left ventricle

The patient's cardiovascular disease has resulted in chronic atrial fibrillation. Her physician has prescribed warfarin (Coumadin®) MOST likely to prevent

A. st!"Oke. B. hypertension. C. atherosclerosis. D. sudden cardiac death. E. left ventricular hypertrophy.

The bone resorption seen in elderly patients with low dietary calcium is enhanced by which of the following hormones?

A. Insulin B. Estrogen C. Aldosterone D. Thyroid stimulating hormone E. Parathyroid hormone

Administration of a local anesthetic with epinephrine will MOST likely produce which of the following cardiovascular effects?

A. B. C. D.

Increased heart rate Decreased heart rate Increased diastolic blood pressure Decreased systolic blood pressure

© 2005 American Dental Association, Joint Commission on National Dental Examinations. All rights reserved. Reprinted by permission.

Page 20: ASDA Released Papers L-M

386. To minimize pain when biting, the patient moves her mandible to the right and chews only on that side. Which of the following muscles primarily produces this mandibular movement?

A. Right masseter B. Left temporalis C. Right medial pterygoid D. Right lateral pterygoid E. Left lateral pterygoid

387. Which of the following BEST describes the t'esult of continued abrasion of the dentures on the underlying alveolar mucosa?

388.

A. Lining mucosa becoming gingival mucosa B. Lining mucosa becoming masticatory

mucosa C. Gingival mucosa becoming

orthokeratinized D.

E.

Masticatory mucosa becoming parakeratinized Orthokeratinized mucosa becoming parake rati n ized

The patient's mandibular denture is determined to be pressing too far inferiorly, so that contraction of the muscular floor renders the denture unusable. Which of the following muscles is responsible for this circumstance?

A. B. C. D. E.

Mentalis Mylohyoid Geniohyoid Genioglossus Medial pterygoid

389. The new denture teeth have been set up so that the disto-buccal cusp of the mandibular first molar rests in the central fossa of the maxillary first molar. Which of the following BEST describes this occlusal classification?

A. B. C. D.

Class I Class II, division I Class II, division II Class III

Test item #'5 390-400 refer to the following testlet.

A patient with hepatitis C is scheduled for extraction of posterior teeth in the right lower quadrant due to advanced periodontal disease. Tooth #28 is abscessed and draining into medial soft tissue. Tooth #30 shows a large carious lesion and is painful.

390. Infection with hepatitis C results in liver dysfunction, which might decrease the liver's ability to store glycogen. In addition to the liver, which of the following is the other major site of glycogen storage?

A. Brain B. Kidney C. Smooth muscle D. Cardiac muscle E. Skeletal muscle

391. Damage to the liver from hepatitis C infection might impair bilirubin metabolism. Bilirubin is a product of which of the following pathways?

392.

A. Albumin synthesis B. Cholesterol synthesis C. Heme degradation D. Copper degradation

-Death from advanced liver disease caused by hepatitis C infection is PRIMARIL Y due to which of the following?

A. Inhibition of replication B. Inhibition of transcription C. Inhibition of urea synthesis D. Increase in serum glucose levels E. Inhibition of immunoglobulin production

393. What is the MOST likely outcome of this patient's hepatitis C?

A. Development of chronic hepatitis B. Development of fulminant hepatitis C. Development of hepatocellular

D.

E.

carcinoma Asymptomatic carrier state, with normal serum transaminase Resolution, with eventual clearing of the virus

© 2005 American Dental Association, Joint Commission on National Dental Examinations. All rights reserved. Reprinted by permission.

41

Page 21: ASDA Released Papers L-M

42 394. If the patient transmits the hepatitis C virus to

the dentist, then it will MOST likely be via which of the following routes?

395.

A. B. C. D.

Oral-fecal Saliva contamination Inhalation of aerosols Contaminated needle stick

Microbial analysis reveals that the flora of the abscess of Tooth #28 consists predominantly of P. gingiva/is. The inflammation is caused primarily by which component of this microorganism?

A. Exotoxin B. M-protein C. Lipoteichoic acid D. Lipopolysaccharide

396. Which of the following is a possible life­threatening sequela of the abscess of Tooth #28?

A. B. C. D. E.

Allodynia Hyperalgesia Non-vital pulp Referred pain Ludwig's angina

397. Which of the following nerve fibers is primarily responsible for the sharp pain in Tooth #30?

A. B. C. D. E.

Aa AI3 A'6 B C

398. If the patient decides to attempt to save Tooth #30, then it will require root canal therapy. Which of the following best describes the MOST likely presentation of pulp canals in this tooth?

A. B

c

Two canals, one in each root Three canals, two in the mesial root and one in the distal root Three canals, one in the mesial 1'00t and two in the distal root

D. Four canals, two in each root

399. If the patient and dentist elect to extract Tooth #30, sectioning of the tooth will be required. The dentist should be aware of which of, the following furcations on this tooth?

400.

A. B. C D. E.

One Two - mesial and distal Two - facial and lingual Three - mesial, distal, and facial Three - facial, mesial, and lingual

If the mandibular right posterior teeth are extracted, then the maxillary right posterior teeth will likely show signs of

A. attrition. B. abfraction. C. ankylosis. D. mesial drift. E. supraeruption.

WHEN YOU HAVE F!NISHED THIS EXAMINATION BOOKLET, PLEASE COMPLETE THE BRIEF CANDIDATE'S QUESTIONNAIRE.

WHEN YOU HAVE COMPLETED THE QUESTIONNAIRE, TURN IN THE FOLLOWING MATERIALS TO THE TEST ADMINISTRATOR:

1. THE TWO ORIGINAL (TOP) ANSWER SHEETS

2. THE CANDIDATE QUESTIONNAIRE

THE TEST ADMINISTRATOR WILL PROVIDE YOU WITH THE ANSWER KEY FOR THE ENTIRE COMPREHENSIVE PART I PILOT EXAMINATION.

YOU MAY KEEP THE EXAMINATION BOOKLET AND THE CANDIDATE'S copy OF THE TWO ANSWER SHEETS.

THANK YOU FOR YOUR PARTICIPATION IN THIS PROJECT.

© 2005 American Dental Association, Joint Commission on National Dental Examinations. All rights reserved. Reprinted by permission.

Page 22: ASDA Released Papers L-M

3

Item No.

~ 201 202 203 204 205 206 207 208 209 210 211 212 213 214 215 216 217 218 219 220 221 222 223 224 225 226 227 228 229 230 231 232 233 234 235 236 237 238 239 240 241 242 243 244 245 246 247 248 249 250

NATIONAl SOARD COMPREHENSIVE PART 8 P!LOT EXAMSNATION TEST KEY

Discipline Item Key IwmNa. Discipline

Anatomic Sciences B 251 Biochemistry-Physiology

Biochemistry-Physiology C 252 Microbiologv-Pathology

Microbiology-Patholocy D 253 Dental Anatomy & Occlusion

Dental Anatomy & Occlusion C 254 Anatomic Sciences

Biochemistry-Physiology D 255 Microbioloav-Patholooy

Dental Anatomy & Occlusion A I 256 Biochemistrv-Phvsiology

Microbiolooy-Pathology D I 257 Dental Anatomy & Occlusion

Anatomic Sciences C 258 Anatomic Sciences

Biochemistry-Physiology E 259 BiochemistrY-Physiology

Microbiology-Pathology C 260 Biochemistry·Physiology

Dental Anatomy & Occlusion B 261 MicrobioloQv-PatholoQY Microbiology-Pathology A Biochemistry-Physiology D

262 Microbiology-Pathology 263 Biochemistrv-PhysiOiOQy

Anatomic Sciences C 264 Biochemislry-PhVSioiogy

Dental Anatomy & Occlusion B 265 Dental Anatomy & Occlusion Dental Anatomy & Occlusion A 266 Denial Anatomv & Occlusion MicrobioJogy-Patho!OfJ'J E 257 Anatomic Sciences Biochemistry-Physiology D 268 Microbiology-Pathology

Anatomic Sciences C 269 Dental Anatomy & Occlusion Dental Anatomy & Occlusion B 270 Dental Anatomv & Occlusion Dental Anatomy & Occlusion 0 271 Microbiology-Pathology

Microbiology-Patholoav 0 272 Anatomic Sciences Biochemistry-Physiology C 273 Biochemistry-Physiology

Anatomic Sciences B 274 Microbioloav-Pathology Biochemistry-Physiology B 275 Dental Anatomv & Occlusion Anatomic Sciences B 276 Anatomic Sciences Dental Anatomy & Occlusion C 277 Biochemistry.-PhysioJQqV Biochemistry-Physiology D 278 Microbiology-Pathology Microbiology-Pathology C 279 Dental Anatomy & Oc,clusion Anatomic Sciences A 280 Microbiolowt-Patholomr Dental Anatomy & Occlusion 0 281 Biochemistry-Physiology Microbiology-Pathology D 282 Anatomic Sciences Biochemistry-Physiology C 283 Biochemistry-Physiology Anatomic Sciences 8 284 Anatomic Sciences Biochemistry-Physiology C 285 Dental Anatomy & Occlusion Dental Anatomy & Occlusion D 286 Microbiology-Pathology Microbiology-Pathology A 287 Biochemistry-Physiology Anatomic Sciences E 288 Anatomic Sciences Anatomic Sciences A 289 Dental Anatomv & Occlusion Biochemistry-Physiology D 290 Microbiology-Patholoov Microbioloav-Patholoov B 291 Biochemistry-Physiology Dental Anatomy & Occlusion C 292 Anatomic Sciences Anatomic Sciences C 293 Biochem istry-PhVSioloov Microbiology-Pathology E 294 Microbiology-Pathology Dental Anatomy &. Occlusion C 295 Dental Anatomy & Occlusion Anatomic Sciences B 296 Microbiology-Pathology MicrobioloQY-Pathology C 297 Biochemistry-Phvsioloav Biochemistry-Physiology A 298 Anatomic Sciences Anatomic Sciences A 299 Anatomic Sciences Dental Anatomy & Occlusion B 300 Biochemistry-Physiology

Hem Key

C B A E A E A D B D D E C D C D E C E D

--C A B D B C B B A D B E A B A C D B A B A A D E A B E B D C

@ 2005 American Dental Association, Joint Commission on National Dental Examinations. All rights reserved. Reprinted by permisSion.

Page 23: ASDA Released Papers L-M

Item No. ""'"

301 302 303 304 305 306 307 308 309 310 311 312 313 314 315 316 317 318 319 320 321 322 323 324 325 326 327 328 329 330 331 332 333 334 335 336 337 338 339 340 341 342 343 344-345 346 347 348 349 350

NA,'T20NAL 80ARD COMPREHEf'JS5VE PART I PILOT EXAMINATION TEST KEY

Discipline Item Key Item No. Discipline

Microbiology-Pathology A I 351 ,'-','""-" emistry-Physiology Dental Anatomy & Occlusion C 352 Dental Anatomy & Occlusion Anatomic Sciences A 353 Dental Anatomy & Occlusion Biochemistry-Physiology C 354 Anatomic Sciences Microbiology-Pathology A , 355 Biochemistry-Physiology Dental Anatomy & Occlusion A 356 Microbiolomr-Pathology Biochemistry-Physiology E 357 Dental Anatomy & Occlusion Anatomic Sciences E 358 Anatomic Sciences Microbiology-Pathology E 359 Biochemistry-Physiology Dental Anatomy & Occlusion A 360 Microbiology-Pathology Dental Anatomy & Occlusion C 361 Dental Anatomy & Occlusion Microbiology-Pathology C 362 Microbiology-Pathology Biochemistry-Physiology A 363 Biochemistry-Physiology Anatomic Sciences D 364 Biochemistry-Physiology Biochemistry-Physiology C Anatomic Sciences Microbiology-Pathology E 66 Anatomic Sciences Dental Anatomy & Occlusion D 367 ,Anatomic Sciences Dental Anatomy & Occlusion A 368 Dental Anatomy & Occlusion Microbiology-Pathology E 369 Microbiology-PathoJogy Biochemistry-Physiology B 370 Microbiology-Pathology Anatomic Sciences B 371 Microbiology-Pathology Biochemistry-Physiology C 372 Biochemistry-Physiology Microbiology·PatholCl£Y C 373 Anatomic Sciences Dental Anatomy & Occlusion B 374 Dental Anatomy & Occlusion Microbiology-Pathology B 375 Dental Anatomy & Occlusion Biochemistry-Physiology 0 376 Dental Anatomy & Occlusion Anatomic Sciences E 3n Anatomic Sciences Microbiology-Pathology C 378 Microbiology-Pathology Dental Anatomy & Occlusion A 379 Microbiology-Pathology Dental Anatomy_ & Occlusion A 380 Anatomic Sciences Microbiology-Pathology C 381 Microbiology-Pathology Anatomic Sciences E 382 Mlcrobiology-PathoJogy Biochemistry-Physiology B 383 Biochemistry-Physiology Microbiology·Pathology B 384 Biochemistry-Physiology Dental Anatomy & Occlusion C 385 Biochemistry-Physiology Microbiology-Pathology E 38S Dental Anatomy & Occlusion Biochemistry-Physiology B 387 Anatomic Sciences Anatomic Sciences C 388 Anatomic Sciences Mlcroblology-Pathole>gy B 389 Dental Anatomy & Occlusion Dental Anatomy & Occlusion E 390 8iochemistry-Physioloav Anatomic Sciences D 391 Biochemistry-Physiology Microbiology-Pathology E 392 Biochemistry-Physiology Dental Anatomy & Occlusion E 393 Microbiology-Pathology Anatomic Sciences 8 394 Microbiology-Pathology Dental Anatomy & Occlusion B 395 Microbiology-Pathology

Microbiology-Pathology D 396 Anatomic Sciences Biochemistry-Physiology C 397 Anatomic Sciences

Anatomic Sciences D 398 Dental Anatomy & Occlusion

Dental Anatomy & Occlusion D 399 Dental Anatomy & Occlusion

Microbiology-Pathology E 400 Dental Anatomy & Occlusion

Item Key

D E 0 C A B A C B E B C D D B C A E A C D D E C C C E C E B A E A E A E C 8 A E C C A D D E C 8 C E

© 2005 American Dental AssOCiation, Joint Commission on National Dental Examinations. All rights reserved. Reprinted by permission.

4

Page 24: ASDA Released Papers L-M

National Board DentaJ l~xanljnations TEST PACKET I-M

Compiled, released items from approximately 1998-2005

American Student Dental Association 211 hlsl ( Avenue' Chic:l!2(l, Jllinoi, (,0(, II • ~ I ;>·440279:')

I·· 1\1;111: J\SI)A(u'AS! • Weh,;I" wwwASIJ/\J\l'Lm;:

Page 25: ASDA Released Papers L-M

2]] Eas1 Chicago Avenue· Chicago, Illinois 606] J • 3]2-440-2795 • Fax: 3]2-440-2820

Contents

E-Mail: [email protected] • Website: www.ASDAne1.org

TEST PACKET I-M

National Board Dental Examination Part I

(Compiled, released items from

approximately 1998-2005)

Page

Randomly ordered test items ........................................................... 3

Answer Key .................................................................................... 22

Page 26: ASDA Released Papers L-M

2.

3.

Which of the following nerves innervates the

Inferior buccal mucose of the flom of the oral cavity?

A. Facial B. Trigeminal C Lingual D. Mylohyoid E. Hypoglossal

Each of the following structures increases the

suliace area of the small intestine EXCEPT one. Which one is the EXCEPTION?

A Villi B. Rugae C. Microvilli D. Brush border E. Circular folds

The inferior parathyroid gland develops from which of the following structures?

A B C. D. E.

Second pharyngeal arch Fourth pharyngeal arch Fifth pharyngeal arch Third pharyngeal pouch Fourth pharyngeal pouch

4. Which of the following controls parathyroid gland function?

A B. C. D. E.

Thyroxin Calcitonin Blood levels of calcium Blood levels of magnesium Thyroid stimulating hormone

5. The splenic artery is a branch of which of the following arteries?

A B. C. D. E.

Celiac Left hepatic Left gastric Musculophrenic Superior mesenteric

3

6

7.

8

9.

10.

Which of the following muscles partici[Jates in

flexion at tile gleno-humeral and the humero-ulnar

joints?

A Deltoid B. Blachialis C Biceps brachii D. Triceps brachii E. Coracobrachialis

Cell bodies of sympathetic fibers in the nerve of

the pterygOid canal come from which of the

following?

A Facial nerve B. Superior cervical ganglion C. Greater petrosal nerve D. Glossopharyngeal nerve E. Otic ganglion

In the upper limb, which of the following represents a hallmark of lymphatic vessels?

A B. C. D.

Contain valves Follow the veins Always travel in pairs Only found on the anterior suliace of the limb

E. Contain fenestrations to allow passage of fluids into the interstitium

Which of the following nerves is associated wittl the second pharyngeal arch?

A Vagus B. Glossopharyngeal C. Accessory D. Mandibular E. Facial

The muscles of facial expression are derived from which of the following?

A Frontonasal process B. First arch C. Second arch D. Third arch E. Fifth al'ch

Page 27: ASDA Released Papers L-M

11 Each of the following crclilial nerves is associated

with the cavernous sinus EXCEPT one. Which one is the EXCEPTION?

A. Facial B. Abducens C. Trochlear D. TI'igeminal E. Oculomotor

12. Which of the following nerves innervates the taste

buds of the anterior two-thirds of the tongue?

A. B. C. D. E.

Vagus Facial Trigeminal Hypoglossal Glossopharyngeal

13. A deviation of the tongue, when protruded, away from the midline results from damage to which of the following cranial nerves?

A. B. C. D. E.

V VII X XI XII

14. Which of the following exhibits phagocytic activity in the central nervous system?

15.

A. Ependymal cell B. Microglial cell C. Oligodendrocyte D. Fibrous astrocyte E. Protoplasmic astrocyte

Each of the following cell layers is derived from oral epithelium EXCEPT one. Which one is the EXCEPTION?

A. B. C D. E.

Dental follicle Stellate reticulum Stratum intermedium Inner enamel epithelium Outer enamel epithelium

4

16. The nerves of the internal thoracic wall lire

immediately deep to which layer of the wal!?

A. External intercostal B. Internal intercostal C. Transversus thoracis D. Parietal pleura E. Subcostals

17. Which of the following represents the condition in

which a radiograph of a 10-year-old patient's

maxilla shows that two succedaneous teeth are

absent?

A. B. C.

D. E.

Mesiodens Gemination Dens in dente Latent odontogenesis Partial anodontia (hypodontia)

18. Where is the channel of communication between the maxillary sinus and the nasal cavity situated?

19.

A. B. C

D.

In the superior nasal meatus In the sphenoethmoidal recess In the middle nasal meatus, at the semilunar hiatus At the anterior extremity of the inferior nasal meatus

E. At the posterior extremity of the inferior nasal meatus

In the temporomandibular joint, a very dense collection of organized elastic fibers is found in which of the following areas of the articular disc?

A. Anterior band B. Posterior band C. Intermediate band D. Posterior-inferior lamina of the bilaminar

zone E. Posterior-superior lamina of the bilaminar

zone

20. Which of the following represents a muscle that

might assist in depressing tile mandible?

A. Masseter B. Stylohyoid C. Teillporalis D. Geniohyoid E. Medial pterygoid

Page 28: ASDA Released Papers L-M

21. The organic componenl of dentin is comprised

primarily of

/\. B. C o E.

keralin. reticular fibers. type I collagen. tyre II and III collagell. oxytalan fibers.

22 Each of the following structures leaves the cranium

by way of the jugular foramen EXCEPT one. Which one is the EXCEPTION?

A B. C o.

Cranial nerve IX Cranial Ilerve X Cranial nerve XI Cranial nerve XII

E. Sigmoid sinus - internal jugular vein

23. Each of the following venous channels has direct connections with the pterygoid venous plexus

EXCEPT one. Which one is the EXCEPTION?

24.

A MaXillary vein B. Vertebral vein C. Deep facial vein D. Infraorbital vein E. Posterior superior alveolar vein

Stimulation of the lesser petrosal nerve in an adult causes secretion by which of the following glands?

A B. C o. E.

Parotid Lacrimal Sublingual Submandibular Glands of the hard palate

25. Ameloblasts will not differentiate from preameloblasts unless they

A B. C. D. E.

contact dentin are touched by odontoblast processes are touched by stratum intermedium contact stellate reticulum contact neural crest mesoderm

5

26. Cells thai will form the vertebrae have their origin

in which of the following?

A. B. C D. E.

Notochord Neural arch Dermamyotome Two pairs of somites Intermediate mesodermal plate

27. The nasolacrimal duct drains into which of the

following?

A B. C D.

Middle meatus Inferior meatus Superior meatus Sphenoethmoidal recess

28. The carotid sheath encloses each of the following structures EXCEPT one. Which one is the EXCEPTION?

29.

A. Vagus nerve B. Extemal carotid C. Internal jugular vein D. Common carotid artery

Following eruption and initial occlusal contact in

the oral cavity, a tooth will continue to erupt in order to compensate for occlusal wear. In response to this continuous eruption, which of the following is deposited at the apex of the root?

A. B. C. D. E.

Primary dentin Secondary dentin Reparative dentin Cellular cementum Acellular cementum

30. Which of the following cells is most likely to be engaged in mitosis at any given time?

A B. C D. E.

Osteocyte Macrophage Plasma cell Chondrocyte Basal keratinocyte

Page 29: ASDA Released Papers L-M

31. Fibrocanilage normally occurs in

A. 8. C. o E.

epiglottis external ear. epiphyseal plate. inter'venebral discs. C-shaped rings ill the wall of the trachea

32. There is a distinct change in the type of surface

epithelium at the junction of the

A B. C D. E.

oropharynx and esophagus. esophagus and stomach. fundus and pylorus. duodenum and jejunum. ileum and colon.

33. In which of the following would fenestrated

capillary plexi be found?

A B. C. D. E.

Predentin Dental pulp Cellular cementum Periodontal ligament Reduced enamel epithelium

34. Submucosal glands are usually located in the

A B. C. D. E.

fundus of stomach. duodenum appendix. jejunum. colon.

35. Difficulty in raising the shoulder might be the result of damage to which of the following cranial

nerves?

A. V B. VII C. X O. XI E. XII

36. A stab wound creating a pneumothorax on the left

side will usually result in collapse of which of the

following?

A The left lung only B. The right lung only C. Both lungs D. The rib cage on the left side E The lef1lung and pericardial sac

37. Shar'pey's fibers from the periodontal ligament

ins(Ori into which of the following structures?

A B. C. D. E.

Bundle bone and dentin Conical plates and dentin Bundle bone and cementum Conical plates and cementum Haversian bone and cenientum

38. In an adult, a Babinski sign indicates damage to

A B. C. D. E.

the dorsal horn. lower motor neurons. upper motor neurons. the lateral spinothalamic tract. the dorsal spinocerebellar tract.

39. Cell bodies of neurons mediating proprioception from the periodontal ligament of the maxillary first

molar lie primarily in the

6

40.

A B. C. D. E.

semilunar ganglion. motor nucleus of V. spinal nucleus of V. chief sensory nucleus of V mesencephalic nucleus of V

In hydrocephalus, excess cerebral spinal fluid is

found within which of the following?

A. Ventricle B. Subdural space C. Cisterna magna D. Subarachnoid space E. Superior sagittal sinus

41. Intelligence and sensory-motor functions of a patient appear to be intact. However, the patient lacks self-discipline and has exhibited anti-social

behavior. He is unable to plan for the future or to

organize behaviors into logical sequences. A lesion

in which of the following areas is likely?

A. B. C. D. E.

Frontal lobe Occipital lobe Internal capsule Teillporallobe Parietal lobe

Page 30: ASDA Released Papers L-M

42. The neurons of the central nervous system that

innervate muscles derived from branchial arches

are found In which of the following nucle()

A B C o E.

Nucleus ambiguus Hypoglossal nucleus Dorsal motor nucleus of X Nucleus of Edinger-Westphal Superior salivatory nucleus

43. Which of the following represents the basis for the

topical application of fluorides in dental caries pr'evention?

A. Fluoride penetrates the enamel through the lamellae.

B Keratin content of the enamel is made more insoluble.

c

D.

E.

Fluoride coats the tooth forming a uniform protective barrier. The primary cuticle, being less calcified, absorbs the fluoride. Acid solubility of the surface enamel is reduced by the fluoride.

44. Which of the following represents the primary mineral component of alveolar bone in the periodontium?

A Osteoid B C. D. E.

Elastin Collagen Hydroxyapatite Ground substance

45. Which of the following represents the primary function of cementum?

A B. C D E.

Attach Sharpey's fibers Protect the root from caries Repair traumatic injuries of the root Supply nutrition to the periodontal ligament Maintain the width of the periodontal ligament

46. The junction between the tooth surface and the

crevicular epithelium is composed of whiCh of the following?

A Cementoid B. Tight junction C Intermediate filaments D. Basal lamina-like structure E. Interstitial crevicular fluid

7

47. Mucosa of the anterior two-thirds of the tongue

develops primaril)' from

A. B. C D. E.

Rathke's pouch tuberculum impar. foregut endoderm. hypobranchial eminence. lateral lingual swellings.

48. Initial/y, the developing hean is

49.

A. B. C D.

induced by the notochord. positioned posterior to the notochord. positioned anterior to the prochordal plate. between the prochordal plate and the notochord.

Melanocytes migrate to the lamina propria of the oral mucosa from which of the following?

A Myotome B. Dermatome C Sclerotome D. Neural crest E. Rathke's pouch

50. Blood from the cephalic vein drains into which of the following veins?

A B. C. D. E.

Basilic Braciocephalic Axillary Internal thoracic Superior vena cava

51. Coupled respiration requires each of the following EXCEPT one. Which one is the EXCEPTION?

A B. C. D. E.

ADP Oxygen Carbon dioxide Electron donor Inorganic phosphate

52. If the anticodon on transfer-RNA is 5'ACG3', then

which of the following is its corresponding codon on messenger-RNA?

A 5' CGT 3' B. 5' CGU 3' C. 5' TGC 3' D 5' UAG 3' E. 5' UGC 3'

Page 31: ASDA Released Papers L-M

Which of the following is the major storage form of iloll in the-; body~)

A. B. C D. E

Transferrin Hemosiderin Apoferritin Hemoglobin Ferrilin

5-'1. When a mammalian red blood cell is placed in 0.3

per cent sodium chloride, water moves across the cell membrane by which of the following processes?

A B. C D. E.

Osmosis Hemolysis Pinocytosis Active transport Diffusion

55. Free fatty acids in plasma are

A. B. C. D.

metabolically inert. mainly bound to [beta]-lipoproteins. independent of epinephrine secretion. mainly associated with serum albumin.

56. Each of the following is required for normal blood Glot formation EXCEPT one. Which one is the EXCEPTION?

A Ca++ B. Plasmin C. Thrombin D. Vitamin K E. Proteolysis

57. Which of the fOllowing combines with heparin to inhibit blood clotting?

A. B. C D. E.

Anti-thrombin III Platelets Fibrinogen Plasminogen Thrombin

8

5A Each of the following is an anticoagulant EXCEPT one. Which one is the EXCEPTIOI\j~;

/J,. B. C. D. E.

Heparin Vitamin K Dicumarol Sodium citrate Any antithrombin sUbstance

59 The characteristic of the aorta lhat is most

responsible for the maintenance of diastolic blood pressure is its

A B. C. D. E.

wide lumen. elastic distenSibility. proximity to the heart. great peripheral resistance. active contraction.

60. Colloid osmotic pressure of the blood is important because it

A B. C. D. E.

nourishes blood cells. aids in blood clotting. prevents loss of erythrocytes from the blOod. prevents excess loss of fluid from capillaries. prevents entrance of tissue fluid into capillaries.

61. Which of the following is most likely to promote de polymerization of extracellular matrix?

A. Cortisone B. Collagenase C. Chymotrypsin D. Hyaluronidase

62. Which portion of the nervous system contains the cardiac, vomiting, and vasomotor centers?

A B. C D.

Medulla Thalamus Cerebrum

Cervical region of the spinal cord

Page 32: ASDA Released Papers L-M

64

The concentration of which of the following amino

acids can be used to determine whether or not

dentin has contaminated a purified enaillel preparation?

A. B. C. o. E.

Lysine Proline Hydroxyproline Phosphotyrosine Phosphoserine

Calcium is transported in the blood in combination with which of the following?

A. AlbUmin B. Citrate C. Heillogiobin D. Beta-globulin

65. Sympathetic stimUlation most likely produces which of the following?

A. B. C. D. E.

Glycogen synthesis Bronchial dilation Decreased mental activity Decreased metabolic rate Increased blood flow to the skin

66. Which of the following is the same value for intracellular and interstitial fluid?

A. [Na+]

B. [Cn

c. [K+]

D. Colloid osmotic pressure E. Total osmotic pressure

67. Which of the following explains why an increased filling of the ventricle during diastole causes a

more forceful heali beat? The increased ventricular volume

A. dimin'lshes the refractory period of the ventricle.

B. increases end-diastolic fiber length. C. facilitates conduction in the heart. D. produces a less than optimal load.

9

68. Prior to surgery an anxious patient has a higher

systoliC blood pressure than previously noted.

Which of the following represents the most likely reason';

A. B. C. D. E.

Cardiac shock Anaphylactic shock Atrial fibrillation Ventricular fibrillation Decreased arterial compliance

69. Reduced renal blood flow can cause hypertenSion

by which of the following?

A. Release of renin B. Reflex vasoconstriction C. Retention of potassium in blood D. Increased output of epinephrine

70. Which of the following describes the effect of a

drug that inhibits renal carbonic anhydrase?

A. B. C. O.

It decreases urea clearance. It increases the Tm for glucose. It increases the acidity of the urine. It decreases sodium reabsorption in the proximal tubule.

71. The composition of plaque is most similar to which of the following?

A. Bone B. Enamel C. Calculus D. Oral bacteria E. Saliva

72. Which of the following substances is the predominant source of ATP at MODERATE levels (for greater than 60 minutes) of activity?

A. B. C. O.

Amino acids Fatty acids Carbohydrates Proteins

Page 33: ASDA Released Papers L-M

73

74

75.

Which of the following substances has a ciearance

rate that is greatel than the glomerular filtration rate (GFRfJ

A. Para-aminohippuric acid (PAH) B Potassium C Glucose D. Urea

What is the biological Significance of the extensive degeneracy of the genetic code?

A

B.

Alters the amino acid sequence of proteins encoded by the DNA Minimizes the deleterious effect of mutations

C. Maximizes the beneficial effect of mutations O. Increases chain termination E Leads to active proteins

Which of the following types of blotting can be used to identify DNA restriction fragments?

A. B. C. O.

Eastern Southern Northern Western

76. The catabolism of which of the following results in no energy production in the form of ATP?

A B. C. O.

Lipid Protein Nucleotide Carbohydrate

77. Which of the following substances is released by blood platelets and causes platelets to stick together?

A. B. C O. E.

Thrombin Fibrinogen Phospholipids Throillboxane A2 Prostacyclin 12 (PgI2)

10

78. Peptide hormones generally exert their effect

thlOugh which of the iollowlllg'i

A.

B.

C.

O.

Intracellular receptms and stimulating protein synthesis

Receptol's on the cell membrane and stimulating protein synthesis Intracellular receptors and altering the specific activity of certain enzymes Receptors on the cell membrane and altering the specific activity of ceriain enzymes

79. During the period of isovolumetric contraction,

which of the following happens?

A B. C.

o

The semilunar valves are open. The left ventricular pressure is rising rapidly. The aortic pressure is slightly less than the left ventricular pressure. The right ventricular pressure is greater than the left ventricular pressure.

80. Glucose reabsorption in the nephron is usually completed in which of the following?

A. B. C. O.

Distal tubule Loop of Henle Collecting duct Proximal tubule

81. Which of the following represents the correct size

and characteristic of the nerve fibers that conduct

sensory input of pain from the oral-facial region?

A B. C. D. E.

Large diameter, myelinated Small diameter, myelinated Large diameter, unmyelinated Small diameter, unmyelinated Intermediate diameter, myelinated

82. As blood flows through the periodontal melllbr·ane.

t"emendous pressures occur in response to forceful occlusion. Blood flow is teillporar-ily

reduced, but platelets do not aggregate because of the presence of

A. Ca++ B. AOP. C. thrombin. O. thrornboxane A2. E. prostacyclin (PgI2).

Page 34: ASDA Released Papers L-M

8:1. /\n examp\(" 01 synergism ic; the efiectc, 01

C

o

insulin and glucagon on blood glucose estrogen and progester'one on uterine motilrty.

growth hormone and thyroxine on skeletal growth

antidiuretic hormone and aldoster'one on potassiulll exuetion

84. The jaw jerk reflex is an example of which of the following reflexes?

A Load B. Flexor C. Withdrawal D. Dynamic stretch

85. The stretch reflex is an example of which of the fol/owing reflexes?

86.

87.

A Withdrawal B. Nociceptive C. Polysynaptic D. Monosynaptic

A patient bites down rapidly on an unexpected

hard surface while chewing. Cessation of motor

unit recruitment in jaw closing muscles is caused by stimulation of

A 8. C. D.

muscle spindles. mucosal mechanoreceptors. periodontal mechanoreceptors. nociceptors in the dental pulp.

Which of the following statements about norepinephrine is carTee!?

A Causes cardiac acceleration 8. Causes general vasodilation C. Causes vasodilation in vessels of the skin D. E

Has a negative inotropic effect on the heart Is the preganglionic sympathetic neu rotr'ansmitter

11

88. Which of the following molecules causes the

activatiorl of myosin kinase and the contraction of

smooth muscle'l

A. B. C. O. E.

Actin Troponin Calmodulin Calcitonin Cholecalcileml

89. Which of the following is the primary difference

between juxtamedullary and cortical nephrons?

90.

A Renal plasma flow B. Filtration fraction C. Length of the distal tubule D. Length of the proximal tubule E. Length of the thin segment of the loop of

Henle

Which of the following is an essential element found in all cytochromes?

A. Co B. Cu C Fe D. Mg E. Zn

91. Acute cyanide poisoning would most likely lead to which of the following?

A

B.

c.

D.

E.

Increased oxygen concentration in arterial blood Increased carbon dioxide concentration in venous blood Decreased oxygen extraction by peripheral tissues Increased oxygen extraction by peripheral tissues Decreased carbon dioxide concentration in arierial blood

92. Chondroitin sulfate is a major component of which of the following?

A. Hair B. Mucin C. Cartilage D. Bacterial ceil walls E. Blood group substance

Page 35: ASDA Released Papers L-M

93. Which of it-Ie following characterizes hoth activ("

transpori and facilitated diffusion?

A. Hydrolysis of ATP B. Competitive inhibition C Transport bidil-ectionai D Transport against a concentration gradient

94 _ Which of the following has the most effect in

stimulating respiration?

A. Increase in blood pH B. Decrease in arterial p0 2

C.

D. E.

Increase in arte(lal pC02

Decrease in blood pH Decrease 'In arterial pC02

95_ Which of the following proteins is involved with bacte(lal aggregation and subsequent eliminat'lon

from the oral cavity?

A_ S. C. D.

Gustin Statherin Lactoferrin Secretory IgA

96. Thyroid hormones are synthesized from which of the following amino acids?

97.

98_

A. Tyrosine B. Dopamine C. Histidine D. Threonine E. Tryptophan

Which of the following enzymes is responsible for

inactivating catecholamines?

A. Phosphodiesterase B. Monoamine oxidase C. Amino decarboxylase D. Tyrosine hydroxylase

Protein and RNA synthesis occur in each of the

following phases of the cell cycle EXCEPT one.

Wilich one is the EXCEPTION?

A. GO

B. G1

C. S D. G2

E. M

99. Whicll main product of protein nitrogen metabolislTI

is 10und in human urine?

A Urea B. Ammonia

C. Creatine

D. Uric acid

E Creatinine

100. Each of the following is expected to be active

during fatty acid biosynthesis EXCEPT one. Which

one is the EXCEPTION?

A. B. C. D. E.

Tricarboxylic acid cycle Pyruvate dehydrogenase Amino acid catabolism Beta oxidation Glycolysis

101. Which of the following statements best describes the roots of a maxillary first molar?

12

A.

B.

C.

The mesiofacial root usually tips mesially in its apical one-third. The palatal roo\ has a concave facial surface. The palatal root is flattened and concave on its mesial and distal surfaces.

D. The distofacial root is flattened and concave on its facial surface.

102. Which of the following represents the location of the lingual height of contour on the crown of the mandibular second premolar?

103.

A. Middle third B. Occlusal third C. Same third as that tooth's buccal height of

D.

E.

contour Same third as the lingual height of contour on the crown of the maxillary premolars Same third as the lingual height of contour on the crown of the mandibular first premolar

When v'lewed from the facial aspect, the crown of the mandibular first premolar has an occlusal outline that normally exhibits which of the following characteristics?

A. Cusp tip which is offset to the distal B. Cusp tip which is centered mesiodistally C. Disto-occlusal slope which is longer than the

mesio- occlusal slope D _ Mesio-occlusal slope which is longer than

the disto- occlusal slope

Page 36: ASDA Released Papers L-M

~ 04. Two pulp canals are most commonly found in the

A. B. C D E

root of a mandibular central incisor. facial root of a maxillary first premolar distal root of a mandibular first molar. mesial root of a mandibular first molar. mesiofacial root of a maxillary first molar.

105. In the triangle formed by the projection of the orifices of the canals of a maxillary first molar, the

A. B. C D E.

line connecting mesial with lingual is longest. line connecting distal with lingual is longest. line connecting mesial with distal is longest angle at the distolingual canal is obtuse. angle at the mesiofacial canal is obtuse.

106. Which of the following attaches the root surface to the alveolar bone?

A. Cementum B. Attached gingiva C. Dentinal tubules D. Periodontal ligament E. Cementoenamel junction

107. Which of the following anatomic structures is found just lingual to the maxillary central incisors?

A. Incisive foramen B. Canine eminence C. Maxillary tuberosity D. External oblique ridge E. Greater palatine foramen

10S. Which of the following papillae would normally be found in the buccal vestibule?

A. Parotid B. Incisive C Fungiform D. Interdental E. Circumvallate

109. The bulk of a tooth consists of

A. pulp. B. crown. C. dentin. D. enamel. E. cementum.

13

110 The part of the tooth which, at a given moment, is

exposed to the oral cavity is

A. B. C. D

E.

the anatomic crown. the clinical crown. the functional clinical crown and root. measurably larger ill young per'sollS than in older persons. not efiected by periodontal health.

111. The anatomic crown is that portion of a tooth that is

A. B. C. D

in occlusion. exposed to oral fluids. coronal to the cervical line. occlusal to the gingival margins.

112. A patient has an extremely wide, notched tooth in the mandibular left central incisor position. Clinical and radiographic examinations reveal 28 teeth have erupted, but four third molars have not erupted. Which of the following conditions exists?

A. Fusion B. Dens in dente C. Concrescence D. Dilaceration E. Gemination

113. When compared to a maxillary first molar, the roots of a maxillary second molar

A. are longer. B. are more divergent. C. are fewer in number. D. have less potential for fusion. E. are greater in distal inclination.

114. Which of the following teeth is the most likely to have a bifurcated root?

A. B. C. D. E.

Maxillary central incisor Maxillary lateral incisor Mandibular lateral incisor Maxillary canine Mandibular canine

115. Which of the following primary teeth does a second premolar replace when it erupts into the oral cavity?

A. Canine B. First premolar C. Second premolar D. First molar E. Second molar

Page 37: ASDA Released Papers L-M

11 (:, Which of the following describes tile contact relationship between 8 I118xilialY central incisor allo a maxillary lateral incisor?

A B. C

D.

Contact is offset to the lingual Contact is centered incisocervically.

Lingual embrasure is larger than the facial embrasure.

Incisal embrasure is the largest of all the embrasures.

117. In the horizontal plane, as the mandible moves in a lateral excursion, the midline of the mandible moves

A. B. C. D. E.

straight anteriorly. straight posteriorly. straight laterally. anteriorly and laterally. posteriorly and laterally.

118. A patient presents with symptoms of an acute abscess on the maxillary left lateral incisor. There are no clinical signs of decay or restoration. Radiographically, which of the following will most likely be shown as the cause of the abscess?

A. Agenesis 3. Dilaceration C. Concrescence D. Dens in Dente E. Enamel pearls

119. Each of the following can be found in the dental pulp EXCEPT one. Which one is the EXCEPTION?

A. B. C. D.

Nerve tissue Blood vessels Cementoblasts Lymph vessels

120. Which of the following molar roots is wide faciolingually and concave on both mesial and distal surfaces?

A. Distofacial of a maxillary first B. Lingual of a maxillary first C Mesial of a mandibular first D. Distal of a mandibular first

121, The distolingual cusp of which of the following molal's might be absent?

A. B. C D.

Maxillary first Mandibular first Mandibular third Maxillary third

14

122. In a right working movement, the lingual cusp of a maxillal'y rrght second premolar passes through whicll of the following mandibular structures?

A. B. C

D.

FaCial groove of the right first molal Lingual groove of the right first molar Embrasure between the right first premolar and the right second premolar Embrasure between the right second premolar and the right first molar

123, The proximal contact areas between anterior teeth are incisal to the middle third of the teeth. Which 01 the following is an EXCEPTION to this rule?

A B. C.

D.

The mesial contact of a maxillary canine The distal contact of a maxillary canine The mesial contact of a maxillary lateral incisor The mesial contact of a mandibular lateral incisor

124, When the mandible performs a laterotrusive movement, the laterotrusive-side condyle moves primarily about which of the following axes?

A. Vertical B. Sagittal C. Horizontal D. Transverse

125. During a visual examination of a patient who has a severe clenching habit, which muscle would one expect to see enlarged?

A. B. C. 0,

Masseter Temporalis Medial pterygoid Lateral Pterygoid

126. From the occlusal aspect, which of the following represents the most frequently seen form of a maxillary second molar?

A. Round B. Square C. Trapezoidal D. Rhomboidal E. Heal1-shaped

127. Initiation of calcification fm the mandibular central incisor normally OCCUI'S at

A B. C. D. E.

3-4 months of age. 1 year of age. 2-3 years of age. 4-5 years of age. 6-7 years of age.

Page 38: ASDA Released Papers L-M

1 n The oblique ridge of the maxillalY molars extends I)(-:twloell which two cusps?

A B C D. E.

Meslofacial and distofacial Mesiolingual and distolingual Mesiollllgual and distofacial Mesiofacial and mesiolingual Mesiofacial and distolingual

129. At age 8, the maxillary first molar has

A B. C D. E.

a mesial contact with the primary first molal. a distal contact with the second molar. no mesial contact. no distal contact. not yet erupted.

130. Of the primary maxillary teeth, the cervical ridge would stand out most prominently as a distinct entity on which surface of which molar?

Molar Surface

A First Distofacial B. First Mesiofacial C. Second Distofacial D. Second Mesiolingual E. Second Distolingual

131. Which groove of the mandibular first molar does the maxillary mesiolingual cusp pass through in a lateral excursive movement on the working side?

A Buccal B. Lingual C Central D. Distobuccal

132. Which of the following premolars frequently has only one pulp horn?

A B. C D.

Maxillary first Mandibular first Maxillary second Mandibular second

133. The illcisal embrasure is the smallest between which of the following two teeth?

A. Maxillary central incisors B Mandibular central incisol's C Maxillary central and lateral incisors D. Maxillary lateral incisor and canine E Mandibular lateral incisor and central inciSal

15

134. When in its normal position relative to the arch fornl thce: crown of a mondibular first 1110101 inclines.

A. B. C. D.

distally and facially. mesially and facially. mesially and lingually distally and lingually.

135. Which of the following teeth mighl possess three cusps?

A

B.

c.

D.

E.

Maxillary second pl'emolar and maxillary first molar Maxillary second premolar and mandibular first premolar Maxillary first molar and mandibular second molar Maxillary second molar and mandibular first premolar Maxillary second molar and mandibular second premolar

136. Each of the following grooves originates in the central pit of the maxillary second molar (four cusp type) EXCEPT one. Which one is the EXCEPTION?

A. B. C. D.

Buccal Central Distolingual Transverse grooves of oblique ridge

137. When the mandible moves from maximum intercuspai position distally, any tooth contacts that occur are called

A retrusive contacts. B. protrusive contacts. C. mediostrusive side contacts. D. laterotrusive side contacts. E. lateral protrusive contacts.

138. Between which of the following permanent teeth is the lingual embl'3sure smaller than the facial embrasure?

A

B.

c.

D

Maxillary first premolar and maxillary second premolar Maxillary second molar and maxillal)! third molar Mandibular first molar and mandibular second molar Mandibular first premolar and mandibular second premolar

Page 39: ASDA Released Papers L-M

139 On a mandibular first molar, the distofacial groove serves as an escapeway for the mesiolingual cusp of the maxillary first molar during which of the following mandibular movements?

A. Working B. Non-working C. Protrusive D. Centric slide

140. Which of the following primary grooves uniting in the distal pit on the occlusal surface of the mandibular second molar represents the one that normally has no counterpan in the distal pit of the first molar?

A. Distolingual B. OF triangular C. DL triangular D. Distal marginal E. Distal ponion of the central

141. Which of the following best describes the cervical margin on the facial crown surface of the maxillary first molar?

A. Straight B. Evenly convex toward the apex C. Evenly convex toward the occlusal D. Irregularly convex toward the apex E. Irregularly convex toward the occlusal

142. Which of the following represents the most common type of root anomaly on the mandibular first premolar?

A. Dwarfing B. Elongation C. Bifurcation D. Concrescence E. Trifurcation

143. Which of the following teeth represents the one most likely to present with three roots?

A. Mandibular central incisor B. Mandibular second premolar C. Mandibular canine D. Maxillary first premolar E. Maxillary second premolar

16

144 Which of the following represents the structure in the maxillary alveolar bone Hlat maxillary premolar roots occasionally penetrate?

A. Antrum B. Nasal septum C. Frontal sinus D. Zygomatic arch E. Mandibular fossa

145. Which of the following jaw positions is determined almost exclusively by tooth contact?

A. Rest position B. Terminal hinge C. Maximum opening D. Maximum protrusive E. Intercuspal position

146. In a CUSP TO FOSSA contacting relationship in intercuspal position, the maxillary first premolar is most likely to artiCUlate with which of the following mandibular teeth?

A. Lateral incisor B. Lateral incisor and canine C. First premolar D. Canine and first premolar E. First premolar and second premolar

147. Which of the following is least likely to contribute to or affect stability of the dental arch form?

A. B. C. D. E.

Periodontal health Plane of occlusion Occlusal contact forces Interproximal contact form Forces exened by the lips and tongue

148. When viewed from the frontal plane and progressing posteriorly, the axial inclination of the crowns of maxillary posterior teeth

A remains vertical. B. inclines lingually. C. inclines distally. D. inclines mesially. E. inclines buccally.

149. When viewed from the sagittal plane, the axial inclination of the anter·ior teetll

A. remains venical. B. inclines facially. C. inclines mesially. D. inclines distally. E. inclines lingually.

Page 40: ASDA Released Papers L-M

15CJ The presence of m3melons on a 19-year-old pCltient suggests which of the following conditions')

151.

A. B. C. D. E.

Fluorosis !Vialformation !Vialnutrition Delayed el-uption Anterior open bite

In osteomalacia, bones are weak because of the failure of

A. remodeling of bone, B, bone matrix formation, C osteoblast proliferation D, bone matrix calcification, E, interstitial growth of cartilage,

152, Which of the following is the most common location for an atherosclerotic induced aneurysm?

A. Thoracic aorta B, Abdominal aorta C. Coronary arteries D_ Arch of the aorta E, Common iliac arteries

153, Each of the following statements about poliovirus infections is correct EXCEPT one, Which one is the EXCEPTION?

A, Most infections are subclinical, B_ Paralysis is an uncommon outcome of

infection, C, There are 3 types of poliovirus, making 3

infections possible, D, Virus is predominantly shed from the body

and transmitted in respiratory secretions, E_ Some damaged neurons may be repaired,

restoring lost functions,

154 - Which of the following can result if an individual having reactivation of latent varicella zoster virus transmits virus to a seronegative individual?

A. Shingles B. Chickenpox C. Herpes labialis D, Infectious mononucleosis E, Herpetic gingivostomatitis

155. Which of the following conditions might be an initial manifestation of early, acute HIV infection?

A. Kaposi's sarcoma S, Wasting syndrome C. Oral hairy leukoplakia D_ Mononucleosis-like syndrome E Pneumocystis car-inii pneumonia

17

156, Which of the following pathogens is the single most common cause of sexuCllly transmitted disease in the U_S,?

A. B_ C D, E,

Treponema pallidum Haemophilus ducreyi Chlamydia trachomatis Herpes simplex virus Human immunodeficiency virus

157, Which of the following represents the mechanism of action of diphtheria toxin?

A. Activates cAMP B_ Causes cytolysis C, Inhibits translation D, Inhibits transcription E, Inhibits DNA replication

158, Each of the following fluids is considered one that can transmit HIV EXCEPT one, Which one is the EXCEPTION?

159_

A. Semen S_ Serum C. Saliva D_ Amniotic fluid E, Breast milk

Which of the following represents the anaerobic organism that is cultured from gingival scrapings and that forms black colonies on hemin-containing culture media?

A. Sarcina lutea B_ Bacillus anthracis C_ Veillonella alcalescens D, Porphyromonas gingival is E_ Alpha-hemolytic streptococcus

160, Beta 1-4 linkages connect N-acetyl glucosamine and N-acetyl muramic acid, The resulting polymer is found in bacterial

A flagella B, capsules_ C, cell walls, D, metachromatic granules_

161, Rough pneumococci that are grown in the presence of DNA from smooth pneumococci develop capsules, This process is ter-Illed

A_ translation, B_ tranSduction, C_ transformation, D, conjugation,

Page 41: ASDA Released Papers L-M

1G2 During an outbreak of gastroenteritis caused by Salmonella, a strain of the species suddenly appears to be resistant to several antibiotics Which of the following best explains why?

A. Parent strain has undergone several mutations

B. Parent strain has acquired a plasmid C There are several species of Salmonella

present, each resistant to one antibiotic D. New strain is a species of Escherichia coli,

which has acquired virulence by mutation.

163. Which of the following karyotypes is found in Turner syndrome?

A. 45, XO B. 45, YO C. 46,XX D. 47,XXY E. 47, XYY

164. Which of the following viruses are frequently associated with eye infections?

A. Mumps and measles viruses B. Parainfluenza and rubella viruses C. Coxsackievirus and rhinoviruses D. Adenoviruses and reoviruses E. Herpes simplex virus and adenoviruses

165. In humans, the mumps virus is transferred by

A food. B. feces. C. flies. D. saliva.

166. Human leukocyte antigen (HLA) Class I molecules are found on which of the following?

167.

A All nucleated cells B. CD4+ T cells only C. Epithelial cells only D. HLA Class 11+ cells only E. Mesencllymal cells only

Soft and hard tissue necrosis char'acterizes which of the following fungal diseAses?

A. Mucormycosis B. Cryptococcosis C. Histoplasmosis D. Coccidioidomycosis E. Candidiasis

18

168. Nongonococcal urethritis is ofien caused by microorganisms of whicll of the following genera?

A. Chlamydia B. Treponema C Neisseria D. Hemophilus

169. Whicll of the following is least likely to produce acute abdominal symptoms?

A. Cholelithiasis B. Acute pancreatitis C. Ampulla of Vater cancer D. Carcinoma of the tail of the pancreas

170. Which of the following represents the arthritis that is usually associated with aging?

A. B. C. D.

Osteoarthritis Gouty arthritis Rheumatoid arthritis Psoriatic arthritis

171. Multiple, lytic lesions of bone characterize each of the following conditions EXCEPT one. Which is the EXCEPTION?

A. Multiple myeloma B. Metastatic carcinoma C. Osteogenesis imperfecta D. Hyperparathyroidism E. Langerhans (eosinophilic) granulomatosis

172. Osteoporosis can be associated with each of the following EXCEPT one. Which is the EXCEPTION?

A. Prolonged corticosteroid administration B. Prolonged immobilization C. Chronic malnutrition D. Hypervitaminosis 0 E. Advanced age

173. In general, enzyme-deficiency diseases are inherited by which of the following modes?

A B. C. D.

Polygenic X-linked dominant Autosomal recessive Autosomal dominant

Page 42: ASDA Released Papers L-M

174 Which of the following anemias results from dl'ug. IIlduced bone-: malTOW suppression';

A. Aplastic B. Sickle cell C Pernicious D Hemolytic E. Myelophthisic

175. Multiple drug resistance is associated with

A. B. C. D. E.

plasmids recombination. point mutations. specialized transducing phages. generalized transducing phages

176. Which of the following statements best describes bacterial transformation?

A. DNA coding for RNA synthesis 8. RNA coding for protein synthesis C. Acquisition of an inheritable trait by bacteria

mediated by DNA

D. Acquisition of an inheritable trait by bacteria mediated by RNA

E. Binding of 30S and 70S ribosomes

177. Plasmid-mediated antibiotic resistance has been observed in diseases caused by each of the following EXCEPT one. Which one is the EXCEPTION?

A. B. C D. E.

Streptococcus pyogenes Staphylococcus aureus Neisseria gono1717oeae Borde/el/a pe/tussis Haemophilus influenzae

178. Each of the following can produce a genotypic change EXCEPT one. Which is the EXCEPTION?

179.

A. 8. C. D.

Conjugation Lysogenization Transformation Exposure to tetracycline

TIle human immunodeficiency virus preferentially infects which of the following cells?

A. NK 8. Helper I C Cytotoxic T D. Suppressor T

19

180. Which of the following viruses is most likely to be isolClted from feces?

A. Rubella B. Hepatitis C C Influenza A D. Coxsackievirus E. Herpes simplex

181. Cellular tropism by viruses is dependent on whiCh of the following?

182.

A. B. C. D. E.

Host cell DNA homology to viral DNA Temperature of host cell incubation Gamma-interferon production Cell surface receptors Viral enzyme synthesis

A 5-year-old child with vesicular lesions limited to the palate and the posterior oropharyngeal mucosa

has an oral temperature of 101 0 F (380C). The most probable diagnosis is

A. chickenpox. B. herpangina. C. recurrent oral herpes. D. herpetic gingivostomatitis E. hand-foot-and-mouth disease.

183. Which of the following cells are the most radiosensitive?

A. B. C. D. E.

Neurons Fibroblasts Lymphocytes Chondrocytes Epithelial cells

184. During repair, which of the following substances is essential for precollagen fibers to transfol-m into collagen fibers?

A. B. C. D. E.

Cortisone Carotene Ascorbic acid Prothrombin Thromboplastin

185. Which of the following conditions represents an intOXication rather than an infection?

A. Anthrax B. Botulism C. Chancroid D. Bacteroidosis E. Salillonellosis septicemia

Page 43: ASDA Released Papers L-M

1St! The tensile strength of a healing mucosal wound depends on Wllich of the following')

A B. C. D. E.

Wound hormones Epithelial regeneration Formation of collagen fibers Activation of fibrinolysis Formation of elastic fibers

187. The chemotactic accumulation of leukocytes at the site of immune complex deposition is a result of

A steroids. B. histamine. C. complement. D. antihistamines.

188. Which of the following has the least ability to regenerate?

A B. C. D. E.

Bone Liver Striated muscle Collagen Smooth muscle

189. Healing is retarded by each of the following conditions EXCEPT one. Which one is the EXCEPTION?

A Trauma B. Immobilization C. Infection D. Hemorrhage E. Ischemia

190. Escape of plasma from capillaries into an area of inflammation is favored by an increase in which of the following?

A Hyaluronidase B. Leukocyte migration C. Vascular permeability D. Osmotic pressure of plasma E. Number of endothelial junctions

191. Each of the following conditions predisposes a patient to develop cancer EXCEPT one. Whicll one is the EXCEPTION?

A Asbestosis B. Anthracosis C. Hepatitis C D. Gardner'S syndrome E. Ulcerative colitis

20

192. Metastatic calcifications are most likely to be the result of

A B. C. D. E.

gallstones. atherosclerosis. hyperparathyroidism. osteogenic sarcoma. lymphatic spread of lung cancer.

193 Difficulty in swallowing refers to

A dyspnea. B. achalasia. C dysphagia. D. hemetemesis.

194. Each of the following viruses is potentially capable of causing cell transformation EXCEPT one. Which one is the EXCEPTION?

A. B. C. D. E.

Retrovirus Herpesvirus Picornavirus Hepatitis B virus Human papillomavirus

195. Which of the following terms refers to the presence of digested blood in the stool?

A. Hemosiderosis B. Hemochromatosis C. Hematoma D. Icterus E. Melena

196. Chlamydia trachomatis might cause each of the following diseases EXCEPT one. Which one is the EXCEPTION?

A. Trachoma B. Lymphogranuloma venereum C. Inclusion conjunctivitis D, Non-gonococcal urethritis E. Primary atypical pneumonia

197. Which of the following conditions is most commonly associated with acute pancreatitis?

A. Chronic alcohol abuse B. Diabetes mellitus C. Physical trauma D. Viral infection E. Hypercalcemia

Page 44: ASDA Released Papers L-M

198 Each of the following is a!!ributable to hepatic failure EXCEPT one. Which one is the EXCEPTION?

A Tremor B Gynecomastia C Mallory bodies o Hypoalbuminemia E. Spider telangectasia

199. Red hepatization refers to which of the following?

A. Early stage of acute hepatitis B. Late stage of acute hepatitis C. Cirrhosis caused by hemochromatosis o Congestion of the liver caused by chronic

right sided heart failure E. Stage of lobar pneumonia

200. Acute passive congestion often accompanies which of the following'!

A. Edema B. Inflammation C. Contusion O. Hematoma

21

Page 45: ASDA Released Papers L-M

NATIONAL BOARD PART 1 RELEASED ITEM KEY ITEMS 101-200

101 B 126 0 151 0 102 B 127 A 152 B 103 C 128 C 153 0 104 0 129 0 154 B 105 A 130 B 155 0 106 0 131 B 156 C 107 A 132 B 157 C 108 A 133 B 158 C 109 C 134 C 159 0 110 B 135 E 160 C 111 C 136 C 161 C 112 E 137 A 162 B 113 E 138 0 163 A 114 E 139 B 164 E 115 E 140 B 165 0 116 C 141 0 166 A 117 0 142 C 167 A 118 0 143 0 168 A 119 C 144 A 169 0 120 C 145 E 170 A 121 0 146 C 171 C 122 0 147 B 172 0 123 B 148 E 173 C 124 A 149 B 174 A 125 A 150 E 175 A

23

176 C 177 A 178 0 179 B 180 0 181 0 182 B 183 C 184 C 185 B 186 C 187 C 188 C 189 B 190 C 191 B 192 C 193 C 194 C 195 E 196 E 197 A 198 C 199 E 200 A

Page 46: ASDA Released Papers L-M

NATIONAL BOARD PART 1 RELEASED ITEM KEY ITEMS 1-100

1 B 26 0 51 C 2 B 27 B 52 B 3 0 28 B 53 E 4 C 29 0 54 A 5 A 30 E 55 0 6 C 31 0 56 B 7 B 32 B 57 A 8 8 33 8 58 8 9 E 34 8 59 8 10 C 35 0 60 0 11 A 36 A 61 0 12 8 37 C 62 A 13 E 38 C 63 C 14 8 39 E 64 A 15 A 40 A 65 8 16 8 41 A 66 E 17 E 42 A 67 8 18 C 43 E 68 E 19 E 44 0 69 A 20 0 45 A 70 0 21 C 46 0 71 D 22 0 47 E 72 B 23 8 48 C 73 A 24 A 49 0 74 8 25 8 50 C 75 B

76 C 77 0 78 0 79 B 80 0 81 0 82 E 83 C 84 0 85 0 86 C 87 A 88 C 89 E 90 C 91 C 92 C 93 B 94 C 95 0 96 A 97 B 98 E 99 A 100 0

The above key may be used to determine correct and incorrect answers; however, for this item set, it is not possible to convert a raw score number correct to a standard score or

associated pass/fail status.

22